Biology > QUESTIONS & ANSWERS > BSC 1011L General Biology II Lab Exam 4 (LATEST FALL 2020/2021)/ DOWNLOAD TO SCORE AN A. GUARANTEED  (All)

BSC 1011L General Biology II Lab Exam 4 (LATEST FALL 2020/2021)/ DOWNLOAD TO SCORE AN A. GUARANTEED PASS

Document Content and Description Below

Chapter 43: The Nervous System 1. In vertebrates, the nervous system is composed of the brain and the spinal cord. A. peripheral B. central C. somatic D. autonomic E. sympathetic 2. ... What are the branched extensions of a neuron that receive signals called? A. axons B. cell bodies C. dendrites D. oligodendrocytes E. synapses 3. What is the elongated extension of a neuron that nerve impulses travel along? A. axon B. cell body C. dendrite D. oligodendrocyte E. synapse 4. What are examples of cells that support neurons both structurally and functionally? A. dendrites and nodes of Ranvier B. neuroglia and nodes of Ranvier C. Schwann cells and dendrites D. dendrites, nodes of Ranvier, and neuroglia E. Schwann cells, oligodendrocytes, and other neuroglia 5. You dissect the brain of a mutant mouse and find that parts of the "white matter" look gray. What is a likely cause? A. defects in myelination B. peripheral nerve damage C. oversized dendrites D. excess myelination E. excess neuronal proliferation Nodes of Ranvier are the spaces between sections of axon enwrapped with myelin. Because myelin creates an insulating layer that prevents ion flow, the progress of the action potential "jumps" from node to node, thus speeding up transmission down the axon. 6. You are studying a gene which, when mutant, causes oligodendrocytes to enwrap shorter sections of axon, so that nodes of Ranvier are more closely spaced. What is the likely effect on nerve impulses? A. They will travel faster. B. They will travel slower. C. They will increase in amplitude. D. They will decrease in amplitude. E. They will travel laterally across connected axons. 7. What other cells should you examine for effects from this mutant gene? A. prefrontal neurons B. gustatory neurons C. cholinergic neurons D. Schwann cells 8. Rapid inward diffusion of Na+ produces a dramatic change in membrane potential. What is this event called? A. hyperpolarization B. excitatory postsynaptic potential C. saltatory conduction D. monosynaptic reflex E. depolarization 9. Nerve impulses are electrical signals produced by which structure? A. lipid sheath of Schwann cell B. plasma membrane of neuron C. synapse D. neuromuscular junction 10. The membrane of a resting neuron is most permeable to which of the following ions? A. Cl- B. Ca2+ C. Na+ D. K+ E. H+ 11. When neurons are not producing electrical signals, there is still a voltage difference across their membranes. What is this voltage called? A. channel-gate bias B. electrolyte voltage C. equilibrium potential D. resting potential E. ion current 12. Which of the following characteristics are true for an action potential? I-A threshold potential must be exceeded for an action potential to occur. II-A stimulus either produces a full action potential or none at all. III-During the refractory period it is less likely that stimuli can produce another action potential. A. I only B. I and II C. II and III D. I and III E. I, II, and III 13. What is the process in which impulses jump from node to node? A. facilitated transmission B. refractory action C. incomplete transfer D. repolarization E. saltatory conduction 14. What determines the direction of the voltage change that occurs at the postsynaptic membrane? A. the type of neurotransmitter B. the type of second messenger system used C. the type(s) of ions that travel through opened channels D. the number of ion channels on the postsynaptic membrane 15. At myelinated areas of the axon (i.e. between nodes of Ranvier), which of the following statements is true? A. There is an increased number of voltage-gated sodium channels. B. There is increased resistance across the membrane. C. There is increased capacitance across the membrane. D. There is increased transport of potassium across the membrane. 16. Which ion channel is primarily responsible for the action potential? A. the voltage-gated sodium channel B. the voltage-gated potassium channel C. the ligand-gated sodium channel D. the ligand-gated potassium channel E. the non-gated sodium channel 17. What diffuses across the narrow synaptic cleft between the presynaptic axon and the postsynaptic cell to transmit a nerve impulse? A. neurotransmitter B. electrical impulse C. Na+ ion D. vesicle E. action potential 18. Which neurotransmitter is released from a motor neuron at the neuromuscular junction? A. acetylcholine B. GABA C. ephinephrine D. dopamine E. serotonin 19. When acetylcholine stimulates the opening of ligand-gated ion channels on a postsynaptic cell, what is this event called? A. neuromuscular junction B. temporal summation C. inhibitory postsynaptic potential (IPSP) D. excitatory postsynaptic potential (EPSP) E. synaptic integration 20. What is the name for the gap into which neurotransmitters are released? A. gap junction B. synaptic cleft C. postsynaptic membrane D. presynaptic membrane E. impulse channel 21. Place the following events that occur at the vertebrate neuromuscular junction in the correct order. I-Depolarization of the muscle fiber membrane. II-Generation of an action potential in the neuron. III-Stimulation of Ca2+ entry into the neuron. IV-Release of acetylcholine into the synapse. A. I, II, III, IV B. III, II, IV, I C. I, III, IV, II D. IV, II,III, I E. II, III, IV, I 22. Which of the following are example of biogenic amines? A. epinephrine and endorphins B. GABA and serotonin C. epinephrine, dopamine, and GABA D. epinephrine, dopamine, and serotonin E. epinephrine, norepinephrine, dopamine, and serotonin 23. Place the following events involved in drug addiction in the correct sequence. I- The synapse becomes less sensitive, and normal function can only continue in the presence of the drug. II- CNS responds to increased firing by decreasing the number of drug receptors on the cell membrane. III- Drug molecule prevents receptor endocytosis and causes overstimulation of the postsynaptic cell. A. I, II, III B. III, II, I C. II, III, I D. III, I, II 24. Which of the following drugs has been discovered to bind to acetylcholine receptors? A. serotonin reuptake inhibitors B. thorazine C. cocaine D. codeine E. nicotine 25. The hippocampus, amygdala, and hypothalamus are the major components of the system. A. sympathetic B. reflex C. parasympathetic D. limbic E. autonomic 26. The spinal cord is enclosed by the vertebral column and layers of protective membranes. What are these membranes called? A. neural net B. white matter C. dorsal root D. meninges E. viscera 27. Damage to the substantia nigra, a specific region of the basal ganglia, can produce the resting muscle tremors characteristic of what disease? A. epilepsy B. Parkinson disease C. seizures D. Alzheimer disease E. insomnia 28. A region of the brain receives both sensory information from ascending nerve tracts and motor commands from the cortex and cerebellum. This region plays an important role in motor control. Which region is it? A. basal ganglia B. pituitary gland C. parietal lobe D. thalamus E. medulla oblongata 29. Which area of the cerebral cortex is important for the formulation of thoughts into speech? A. Broca's area B. the pons C. the medulla oblongata D. Wernicke's area E. the cerebellum 30. Which animal phylum lacks a network of nerve cells for gathering information from the environment? A. Porifera - sponges B. Cnidaria - jellyfish, sea anenomes C. Platyhelminthes -- flatworms D. Nematoda - roundworms E. Mollusca - octopus, squid, clams, snails, and other mollusks 31. In early vertebrates, which component of the brain was devoted largely to coordinating motor reflexes? A. thalamus B. forebrain C. midbrain D. hindbrain E. medulla oblongata 32. Where did information processing become increasingly centered in terrestrial vertebrates? A. thalamus B. forebrain C. midbrain D. hindbrain E. medulla oblongata 33. Which region of the CNS listed below is NOT correctly paired with its function? A. Broca's and Wernicke's areas—language and speech B. Hypothalamus—regulates body temperature, appetite, and sleep C. Cerebellum—relay station for ascending and descending tracts D. Corpus callosum—connects the two brain hemispheres E. Hippocampus—memory 34. Which major regions compose the contemporary vertebrate brain? A. rhombencephalon and prosencephalon only B. mesencephalon and rhombencephalon only C. prosencephalon and metencephalon only D. metencephalon, rhombencephalon, and prosencephalon E. rhombencephalon, mesencephalon, and prosencephalon 35. Which of the following structures is NOT a component of the hindbrain? A. pons B. medulla oblongata C. ascending tracts D. cerebellum 36. What is the primary function of the hindbrain in vertebrates? A. temperature control B. integration of sensory data C. memory storage D. control of the heartbeat E. coordination of motor reflexes 37. Where do correlation, association, and learning occur in the brain? A. cerebellum B. corpus callosum C. medulla D. cerebrum E. pons 38. What is the site of primary sensory integration in the brain? A. medulla oblongata B. cerebrum C. cerebellum D. thalamus E. corpus callosum 39. What region of the brain integrates visceral activities, body temperature, and heartbeat? A. medulla oblongata B. cerebrum C. hypothalamus D. thalamus E. corpus callosum 40. Which of the following is NOT a feature of the limbic system? A. contains hippocampus and amygdala B. resides deep within the cerebrum C. responsible for emotional responses D. important in the formation and recall of memories E. part of the hypothalamus 41. The doctor informs you that your grandmother's stroke has caused damage to Wernicke's area. Which activity is likely to become more difficult for her? A. walking B. doing laundry C. having a conversation D. sleeping E. mathematical calculations 42. What additional factor is required to mediate the parasympathetic effects of acetylcholine? A. G-protein coupled receptor B. GABA C. AMPAR D. habituation E. synaptic integration 43. Which division of the nervous system regulates the activity of smooth muscle, cardiac muscle, and glands? A. antagonistic nervous system B. autonomic nervous system C. sympathetic nervous system D. parasympathetic nervous system E. limbic nervous system 44. Which of the following is not part of the peripheral nervous system? A. sympathetic division of the autonomic nervous system B. spinal cord C. parasympathetic division of the autonomic nervous system D. somatic nervous system E. sensory pathways 45. Which of the following targets is NOT regulated by the autonomic nervous system? A. skeletal muscles B. cardiac muscles C. glands of the body D. smooth muscles E. none of these The PNS comprises the somatic (voluntary) and autonomic (involuntary) nervous systems. The sympathetic division of the autonomic nervous system activates the body for fight-or-flight responses. The parasympathetic division generally promotes relaxation and digestion (or "rest and digest"). 46. A cheetah is chasing a gazelle. What division of the vertebrate nervous system is regulating the stress response in both animals? A. somatic nervous system B. parasympathetic nervous system C. sympathetic nervous system D. parasymbiotic nervous system E. cerebral cortex 47. A cheetah has succeeded in catching a gazelle and has just finished feeding on the carcass. What division of the nervous system has increased in activity? A. somatic nervous system B. parasympathetic nervous system C. sympathetic nervous system D. parasymbiotic nervous system E. cerebral cortex 48. For your birthday, you get a robot-building kit with three parts: 1) a set of gadgets for detecting sound, touch, and light, 2) a set of motors that can move mechanical parts, and 3) a microprocessor that can be programmed to respond to inputs according to predetermined rules. You build a robot car, and find it surprisingly lifelike as it navigates around the room. The three parts of the kit are analogous which parts of the human body? A. 1) autonomic nervous system, 2) somatic nervous system, 3) hypothalamus B. 1) sympathetic pathways, 2) parasympathetic pathways, 3) CNS C. 1) sensory pathways, 2) motor pathways and muscles, 3) CNS D. 1) motor pathways and muscles, 2) sensory pathways, 3) CNS 49. Dr. Robinson tapped on a patient's knee with the little hammer, but failed to see the usual kick of the patellar reflex. Since this reflex only involves a few neurons, what part of the nervous system might the doctor became concerned about? A. parasympathetic nervous system B. central nervous system C. sympathetic nervous system D. peripheral nervous system Charcot-Marie-Tooth is the most commonly inherited peripheral neuropathy, and has no known cure. Patients suffer damage to motor and sensory neurons, leading to degeneration of muscles and gradual loss of motor control. 50. What do the the two sets of neurons damaged in Charcot-Marie-Tooth neuropathy comprise? A. peripheral nervous system B. autonomic nervous system C. somatic nervous system D. parasympathetic nervous system E. central nervous system 51. Genetic studies have found that Charcot-Marie-Tooth may be caused by a mutation in the gene Mitofusin-2 (MFN2), which causes mitochondria to form large clumps. In what area of a neuron are these clumps most likely to cause a problem? A. axon B. synapse C. soma D. dendrite E. myelin The long thin structure of the axon makes it vulnerable to "traffic jams" from clumps of protein or organelles. This can block fast axonal transport to the synaptic terminal. 52. Mutation of the peripheral myelin protein 22 (PMP22) gene, which causes demyelination of peripheral neurons, is a more common cause of Charcot-Marie-Tooth disease. In which cells is the function of the PMP22 gene important for normal function? A. Schwann cells B. fibroblasts C. peripheral neurons D. oligodendrocytes E. astrocytes 53. How can mutations in different genes, functioning in different cells, lead to the same disease? From the following, choose the best responses to complete the sentences: "Physiological function requires that multiple cells work together. Various defects in different cells can impair function, leading to that are indistinguishable at the level of the whole organism. In Charcot-Marie-Tooth disease, motor nerve function can be impaired either by a problem within , or a problem in causing ." A. defects; somatic cells; oligodendrocytes; demyelination B. symptoms; motor neurons; Schwann cells; demyelination C. symptoms; sensory neurons; Schwann cells; fast axonal transport D. genotypes; dendrites; axons; fast axonal transport 54. Subsequent to motor neuron degeneration in Charcot-Marie-Tooth disease, muscles begin to weaken and degenerate as well. What is the likely cause? A. Muscles will atrophy if they don't receive regular stimulation from motor neurons. B. Muscles require healthy Schwann cells for myelination as well. C. Muscles contain large numbers of mitochondria. D. Toxins leak out from the dying motor neurons onto the muscles. 55. Which diagnostic test(s) would you recommend for a patient that you suspect is showing signs of early stage Charcot-Marie- Tooth disease? (Check all that apply.) Culture and protein expression profile of oligodendrocytes EKG to look for rhythmic abnormalities X EMG to test muscle responses fMRI of the frontal lobe of the brain fMRI of the rACC of the brain X Genetic testing for mutations in the PMP22 and MFN2 genes X Nerve biopsy to look for signs of demyelination X Nerve conduction study to test the rate of impulse conduction 56. Your roommate is on an ultra-low fat diet to try to lose weight. Given what you know about structure of the nervous system, what advice would you give him? A. "You should drink large quantities of water, because the brain is mostly water." B. "Good for you! Losing weight by any means necessary can only help your brain." C. "You should eat some healthy unsaturated fats, like those found in fish, olive oil, and nuts, because your brain needs fatty acids to build the myelin sheaths around your axons." D. "You should eat trans fats, from donuts, because 'trans' is short for neurotransmission." 57. In his book The Second Brain, Michael Gershon tells how he was often invited to speak at neuroscience conferences as a "novelty" before study of the enteric nervous system earned respect. To which of the following categories do the neurons controlling gut peristalsis belong? (Check all that apply.) X motor pathways X autonomic nervous system brain X parasympathetic nervous system sympathetic nervous system somatic nervous system The word hyponatremia derives from the words hypo ("under" or "less") and natrium ("sodium"). The word natrium is the basis for the abbreviation Na for sodium. Hyponatremia is a rare condition, but has become of more concern with the rise in amateur runners participating in marathons. Since they run slower, they spend longer on the course and have more opportunity to drink water and sweat out additional sodium. 58. Marathon runners are encouraged to stop at water stations along the course to stay hydrated. However, occasionally a runner drinks too much water and suffers from hyponatremia, or "water intoxication". While water itself is not toxic, drinking too much can upset ion balance in the extracellular space. Based on what you know about ions and nerve signaling, the levels of which important ion are most likely to be disrupted in hyponatremia? A. hydrogen B. potassium C. calcium D. sodium E. oxygen 59. You are an EMT on duty for the marathon. It is an extremely hot day. You say to your partner, "This weather is likely to the risk of hyponatremia, because will cause the athletes to lose ." A. increase, sweating, sodium B. decrease, urination, calcium C. decrease, shivering, potassium D. increase, panting, oxygen 60. What branched structure allows a neuron to take in information from more than one source? A. node of Ranvier B. axon C. myelin D. dendrite 61. What part of the neuron is responsible for most of the "house-keeping" tasks of the cell, including protein synthesis? A. myelin B. axon C. dendrite D. soma E. synapse A typical AA battery has a potential of 1.5V. One can think of the potential in a neuron as having similarities to that of a battery. 62. If you were to compare the resting potential of a neuron to a battery, which way is the battery oriented? A. positive terminal outside of the cell, and negative terminal inside B. positive terminal inside of the cell, and negative terminal outside C. positive terminal pointing toward the axon, negative terminal toward the dendrite D. positive terminal pointing toward the dendrite, negative terminal toward the axon E. both terminals inside the cell soma 63. How does the resting potential of a typical neuron compare to that of a 1.5V AA alkaline battery? A. A typical neuron has a resting potential of -70mV, or about 5% the voltage of the battery. B. A typical neuron has a resting potential of -70V, or about 50x the voltage of the battery. C. A typical neuron has a resting potential of +70mV, or about 5% the voltage of the battery. D. A typical neuron has a resting potential of +70V, or about 50x the voltage of the battery. 64. To process information, postsynaptic neurons often add up simultaneous signals from several presynaptic neurons. Which structure and process allow them to do so? A. axons; temporal summation B. axons; spatial summation C. dendrites; temporal summation D. dendrites; spatial summation 65. A neuron that engages in spatial summation acquires a mutation that causes dendrite overgrowth and promiscuous synapsing. What is the likely outcome? A. abnormally low frequency of action potentials B. summation will not occur C. abnormally high frequency of action potentials D. action potentials with stronger depolarization 66. True or false: neurons are the only electrically-active cells. FALSE Muscles are also formed from electrically-active cells. 67. What important resource do Schwann cells and oligodendrocytes provide to neurons? A. oxygen B. nutrients C. physical strength from microtubules D. Ca2+ currents E. insulating layers of membrane 68. A patient suffers from a disease that slows fatty acid synthesis. What is a likely impact of this disease on nervous system function? A. improved functioning due to a healthier weight B. insufficient myelination and slow axon conduction rate C. improved functioning due to better blood flow D. weaker action potentials with smaller depolarization 69. "Synapses are connections between the axon of one neuron and the dendrite of another neuron, where neurotransmitters transmit a chemical signal." How would you modify this statement to make it more complete? (Check all that apply.) The statement needs no modification. X Synapses can also form between an axon and a cell soma. X Neuromuscular synapses form between the axon of a neuron and a muscle. Synapses often form among the parts of a single cell. X Synapses can be electrical as well as chemical. 70. True or false: When a neuron is not firing, the sodium-potassium pump is inactive. FALSE Even when a neuron is not firing, the pump is maintaining the resting membrane potential by transporting ions that are constantly leaking out through the leak channels. 71. Choose the two primary factors that contribute to the resting membrane potential of a neuron. X K+ leak channels make the cell membrane more permeable to K+. The sodium-potassium pump exports 3 K+ from the cell for every 2 Na+ it imports Na+ leak channels make the cell membrane more permeable to Na+. K+ leak channels cause a higher concentration of K+ outside the cell. X The sodium-potassium pump exports 3 Na+ from the cell for every 2 K+ it imports 72. Is the resting membrane potential of a neuron closer to the equilibrium potential for K+ or Na+? A. K+ B. Na+ C. It is an average of the two. 73. Three-quarters of the substantial amount of energy consumed by the brain is used to run a single molecular machine that maintains the resting potential of neurons. What is this machine called? A. K+ leak channel B. sodium-potassium pump C. acetylcholine receptor D. voltage-gated sodium channel E. voltage-gated calcium channel Palytoxin is incredibly toxic, perhaps the second most dangerous non-peptide toxin. It targets the sodium-potassium pump, locking it open and forming a channel through which the ions can passively flow. Palytoxin is produced by zoanthid cnidarians (soft coral), and there have been incidents of poisoning from zoanthids within marine aquaria tanks in people's homes. It is also thought to be the source of the legend of the Limu make o hana ("Seaweed of Death from Hana"). 74. A Hawaiian legend tells of a village that angered a shark god. In revenge, a "seaweed" began to grow in the tidepools that killed anyone who touched it. In fact, this "limu-make-o-Hana" -- a zoanthid cnidarian -- contains a palytoxin that locks the sodium-potassium pump open, allowing free flow of ions. If a villager suffers palytoxin poisoning, which way will the ions in his neurons flow? A. all ions will flow out B. potassium in, sodium out C. chloride and sodium out, potassium in D. sodium in, potassium out 75. What is the likely impact of this toxin on the resting potential of a neuron? A. The resting potential will require more energy to maintain. B. The resting potential will be more negative. C. It will destroy the ion gradients necessary for the resting potential. D. The resting potential will promote action potentials with a larger depolarization. 76. What is the likely impact of this toxin on action potentials? A. They will occur more frequently. B. They will be stronger and more unpredictable. C. Without ion gradients, action potentials are not possible. D. They will have longer refractory periods. 77. Since there is a larger concentration of potassium ions inside a neuron, why is the inside of the neuron negative (-70mV) relative to the outside? A. Potassium ions carry a negative charge. B. Negatively-charged proteins inside the cell counteract the charge of the potassium ions. C. The negative sign is just a historical convention. D. The high concentration of sodium ions outside the cell makes the inside negative by comparison. 78. Sodium ions are single atoms. Why can't they simply diffuse through the cell membrane's lipid bilayer? A. Since they are charged, ions can't cross the hydrophobic center of the membrane without a channel or pump. B. Sodium ions are covalently attached in groups of three, which are too large to diffuse easily. C. Ions do traverse the membrane easily, but the sodium-potassium pump moves them back across. D. Chelator proteins capture any ions that start to cross the bilayer. 79. If K+ can exit the cell through leak channels, why don't they flow out until there are equal concentrations on both sides of the membrane? A. K+ can act as an enzymatic cofactor, and most K+ ions are bound to cellular enzymes. B. The leak channels allow a very slow leak that cannot keep pace with active import of K+ ions. C. Chelator molecules trap K+ on the inside of the membrane. D. The negative electrical potential inside the cell creates a force that holds the K+ ions in. 80. The outside of a neuron is positively charged at resting potential. Why do K+ ions leave the cell when K+ channels open during an action potential? A. The diffusion gradient is the only factor that affects ion movement. B. K+ ions are actively pumped out by the sodium-potassium pump acting in reverse. C. Voltage-gated K+ channels open when membrane potential reaches +50 mV during the depolarization phase. The diffusion gradient and electrical force cooperate to drive K+ ions out of the cell. D. Negatively charged proteins leave the cell at the onset of the action potential. 81. The outside of a neuron is positively charged at resting potential. Why don't negative ions exit from the cell to normalize charge across the membrane? A. The positive charge outside the cell repels negative ions. B. There are no negative ions inside the cell. C. All of the chloride ions are already outside the cell. D. The negative charge inside the cell is carried by large proteins that cannot diffuse through the membrane. 82. What does the Nernst equation allow us to do? A. Predict the function of a specific type of neuron. B. Calculate the equilibrium potential for an ion. C. Use a computer program to model axonal transmission. D. Calculate the magnitude of depolarization during an action potential. 83. When an action potential begins and sodium channels open, why does sodium rush into the cell? (Check all that apply.) X The inside of the cell is negatively charged. The inside of the cell is positively charged. Chloride ions are waiting to bind to the sodium inside the cell. X The diffusion gradient drives sodium into the cell. Channels use energy to move ions across the membrane whenever they are open. 84. What is the basis of the typical -70mV resting potential in neurons? Rank the factors in order from most important to least important. 3 Ca+2 equilibrium potential 1 K+ equilibrium potential 2 Na+ equilibrium potential 85. What limits the frequency of action potentials? A. chloride ions B. spike amplitude C. nodes of Ranvier D. refractory period 86. Your roommate takes up smoking, and you hate it. Use what you know about nicotine to convince her to quit. (Check all statements that are true.) X Nicotine binds to a receptor in the brain that normally receives acetylcholine. Nicotine evolved to protect the tobacco plant by killing grazing cattle. X The brain adapts to smoking by changing the levels of this receptor. Over time, more drug is required to get the same effect. Nicotine is not addictive, but is still very dangerous. Nicotine is identical to an endogenous ligand in the human body. X Nicotine evolved to protect the tobacco plant by killing insects. 87. A patient of yours is injured in a car accident. The surgeon reports that while there is damage to the dorsal roots, the spinal cord and nerves otherwise appear intact. What functions would you expect to be affected in the patient? A. The patient may lose somatic function, but should retain autonomic function. B. The patient may lose motor function, but should retain sensory perception. C. The patient may lose some sensory perception, but should retain motor function. However, lack of proprioceptive input may make muscle control difficult. D. The patient may lose parasympathetic function, but should retain sympathetic function. They should avoid stressful situations. 88. If sympathetic and parasympathetic neurons both release acetylcholine (ACh), how can they have antagonistic effects? (Check all that apply.) X The target cells of each set of neurons respond differently to the same signal. Each type of neuron secretes ACh bound to a different co-neurotransmitter. X Sympathetic neurons activate the adrenal gland, causing secretion of epinephrine. The neurons release different forms of ACh. Parasympathetic neurons activate the adrenal gland, causing secretion of epinephrine. Botulinum toxin-- A toxin produced by the bacterium Clostridium botulinum. Known for sometimes lethal poisoning from infected meat products. In modern controlled medical or cosmetic use, the very same toxin is known as Botox. Tetanus toxin-- Another very potent neurotoxin produced by Clostridium tetani. Strichnine-- A highly poisonous alkaloid produced by the Strychnos nux-vomica tree, historically used to kill rats and other pests. 89. Botulinum toxin, or "botox", has become popular for cosmetic surgery. The toxin prevents neurosecretory vesicles from docking at the plasma membrane and releasing neurotransmitter. How would this toxin affect the neuromuscular junction? A. Since ACh is not released, the muscles stay in a state of repeated contraction. B. Since ACh is not released, the muscles are not activated to contract and remain relaxed. C. Buildup of ACh causes promiscuous activation of all nearby sensory neurons. D. Buildup of ACh leaks back into the brain, creating a calming effect. 90. You accidentally consume botulinum toxin by eating a contaminated can of sausages. What do you predict will be the most harmful physiological effect? A. slowed action potential conduction B. respiratory failure due to spastic paralysis (constant muscle contraction) C. respiratory failure due to flaccid paralysis (relaxed muscles) D. pupil dilation 91. Another toxin that also blocks neurotransmitter release is tetanus toxin; however, it primarily affects inhibitory interneurons of the spinal cord that negatively regulate motor neurons. Which would be symptoms of tetanus toxin poisoning? A. flaccid paralysis B. spastic paralysis C. slowed action potential conduction D. pupil dilation 92. In contrast to these toxins, strichnine acts postsynaptically, to block glycine from binding to receptors on motor neurons. Would you expect the symptoms of strichnine poisoning to be more similar to those of botulinum or tetanus toxin? A. tetanus toxin B. botulinum 93. How do temporal and spatial summation improve the processing power of the brain? A. Temporal summation allows circadian rhythms to regulate neuronal activity. Spatial summation allows latitude to regulate neuronal activity. B. Temporal summation allows a neuron to fire prolonged action potentials, and spatial summation allows a neuron to synapse onto new targets. C. Temporal and spatial summation both make the neuron more likely to fire action potentials. D. Temporal summation allows a neuron to recognize a persistent signal from a single presynaptic cell, and spatial summation allows a neuron to recognize a broad signal from multiple presynaptic cells. 94. K+ channels in the heart do not have binding sites for ACh, yet parasympathetic release of ACh causes outward flux of K+ from heart cells. How does this occur? A. ACh triggers release of epinephrine from the adrenal glands, which in turn causes K+ flux out of heart cells. B. ACh binds to a G-protein coupled receptor on heart cells, and the G protein alpha subunit binds to a separate K+ channel and causes it to open. C. Parasympathetic neurons activate sympathetic neurons, which directly cause K+ flux from heart cells. D. ACh binds and activates a K+ pump on heart cells. 95. What would happen if the G-protein coupled receptor in heart cells that binds to ACh had a mutation that caused it to bind more tightly to the G-protein alpha subunit? A. G-protein activation and K+ channel opening would happen more quickly. Heart rate may remain abnormally low, impairing response to stress. B. This would lead to K+ flow out of heart cells, causing increased hyperpolarization. C. G-protein activation and K+ channel opening would be slowed. Heart rate may remain abnormally high, especially following stress. D. This would lead to K+ flow into heart cells, causing increased depolarization. 96. How do somatic neurons and autonomic neurons differ? (Check all that apply.) Somatic neurons innervate the body surface, while autonomic neurons innervate internal structures. X Somatic neurons control skeletal muscle, while autonomic neurons control smooth and cardiac muscle. Somatic neurons control exocrine glands, while autonomic neurons do not. X Somatic neurons control their effectors directly, while autonomic neurons use a series of neurons. X Somatic neurons are always excitatory, while autonomic neurons may be either excitatory or inhibitory. 97. Why are mammalian reflexes so fast? (Check all that apply.) They use neurons with extra-wide axons. X They complete the sensory input-to-motor output circuit in only 2-3 neurons. X They do not send signals to the brain and back. They activate a large number of sensory neurons, which speeds things up after the motor neuron performs spatial summation. X There was a selective advantage to evolve rapid responses to certain types of stimuli. Chapter 49: The Circulatory System 1. Why is an open circulatory system less efficient than a closed circulatory system? A. Metabolic wastes cannot be removed from tissues in an open circulatory system. B. There is no heart to pump the blood in an open circulatory system. C. It is more difficult to quickly deliver O2 to specific tissues when needed with an open circulatory system. D. Open circulatory systems lack respiratory pigments that bind to oxygen. E. Blood cannot be used to transport nutrients in an open circulatory system. 2. In 2012, Lance Armstrong was stripped of his record-setting, seven Tour de France titles and banned for life from competitive cycling because of his alleged use of performance-enhancing drugs (PEDs). One of the PEDs Armstrong was accused of taking is erythropoietin (EPO). Why would systematic use of this PED give Armstrong an unfair advantage in a bike race? A. EPO stimulates growth of cardiac and skeletal muscle tissue. B. EPO stimulates the formation of new alveoli in the lungs. C. EPO stimulates formation of new capillary beds in skeletal and heart muscle. D. EPO stimulates the synthesis of myoglobin by muscle cells. E. EPO stimulates red blood cell production by the bone marrow. F. EPO increases cardiac output by stimulating the ventricles to contract more strongly. 3. The liquid portion of blood that contains metabolites, wastes, hormones, ions, and a variety of proteins is called the . A. platelet B. plasma C. eosinophil D. edema E. lymph 4. Leukocytes (white blood cells) account for about of the cells in human blood. A. 0.1% B. 60% C. 1% D. 10% 5. Blood is pumped from the heart into the arterial system, which branches from arteries into fine . A. arterioles B. veins C. capillaries D. venules 6. As compared to the circulatory system of most fishes, the circulatory system of adult amphibians has the advantage of A. a four-chambered heart. B. a single circuit. C. a pocket of the gut that serves as an air bladder. D. partially-separate circuits for gas exchange and systemic delivery of blood. E. complete separation between oxygenated and deoxygenated blood. 7. The benefit provided by all hearts with two atria is A. complete separation of oxygenated and deoxygenated blood. B. not all the blood has to pass through both sides of the heart. C. that blood is pumped into body tissues at high pressure. D. that there are two muscular regions for pumping blood out of the heart to different organs. E. there is an extra place to store blood when the animal is at rest. 8. Mammalian, bird, and crocodile hearts have four chambers with two separate atria and ventricles to ensure different circuits for blood flow. A. two B. three C. four D. eight 9. Jim is suffering from congestive heart failure, in which blood flow is backed up in his lungs and excess fluid is escaping the blood and accumulating in his lung tissue. If untreated, increasing pulmonary edema will lead to death by suffocation. What is the most-likely cause of Jim’s congestive heart failure? A. Jim’s SA node is malfunctioning. B. Contraction of his left ventricle is not keeping up with contraction of his right ventricle. C. Contraction of his right atrium is not keeping up with contraction of his left atrium. D. Jim’s pulmonary semilunar valve is not opening properly. E. Jim’s right atrioventricular valve (tricuspid valve) is not closing properly. 10. A higher protein concentration within the distal portion of capillaries draws water into them through the process of A. osmosis. B. filtration. C. active transport. D. diffusion. E. facilitated diffusion. 11. In birds and mammals, a vestige of the sinus venosus of the fish heart remains as the sinoatrial node, which serves as a(n) A. pacemaker. B. baroreceptor. C. site of erythropoiesis. D. inhibitory node. E. diastole detector. 12. What would be the physiological effect of an increase in the venous return to the heart? A. increased sympathetic stimulation of the heart B. increased secretion of ADH (vasopressin) C. decreased secretion of atrial natriuretic hormone D. increased stroke volume E. decreased cardiac output 13. Changes in the arterial blood pressure are detected by located in the walls of the carotid artery and the arch of the aorta. A. pacemakers B. osmoreceptors C. baroreceptors D. Purkinje fibers E. bundle branches 14. Many animals with open circulatory systems are relatively inactive. However, insects such as bumble bees are quite active. How can insects support high levels of activity with an open circulatory system? A. It does not take much energy to move their jointed exoskeleton. B. Insects have a branching tracheal system. C. Insects have Malpighian tubules that are surrounded by hemolymph. D. Insects are small and do not require a transport system for gases. E. Insects get more energy from glucose breakdown than other animals. Despite having an open circulatory system, insects are able to maintain high levels of activity because they do not rely on their low-pressure circulatory systems for the exchange of respiratory gases. Instead, they have a system of air-filled tubes, called tracheae, which bring respiratory gases directly to and from cells. 15. All of the following are functions of the circulatory system EXCEPT A. oxygen, nutrient, and waste transport. B. blood clotting and immune defense. C. temperature regulation. D. creation of red and white blood cells. E. hormone transport. 16. A bone marrow sample from Patient X has abnormally-low numbers of megakaryocytes. What body process would you expect to be impaired in Patient X? A. defense against foreign microorganisms B. transport of oxygen C. maintaining normal osmotic balance between the blood and the interstitial fluids D. blood clotting following injury to a blood vessel E. temperature regulation by vasoconstriction or vasodilation of superficial blood vessels 17. Which of the following solutes is NOT normally found in the plasma? A. albumin B. metabolites, wastes, and hormones C. ions D. hemoglobin E. globulins 18. Which blood cell type is correctly matched to its function? A. leukocytes—oxygen transport B. leukocytes—blood clotting C. platelets—hematopoiesis D. monocytes—defense against microorganisms E. erythrocytes—immunity 19. You are taking a lab practical exam. One station has a microscope slide of a cross section through an artery and vein. Which vessel is the artery? A. the vessel with a greater overall diameter B. the vessel with the wider lumen (e.g., interior space that contains the blood) C. the vessel lined only with endothelium D. the vessel with valves to prevent backflow E. the vessel with more muscle tissue and elastic fibers in its wall 20. Exchange of gases and metabolites between the blood and tissues occurs in the A. arteries. B. arterioles. C. capillaries. D. venules. E. veins. 21. When you are upright, what is the primary mechanism that propels blood in your feet back to your inferior vena cava? A. recoil of elastic fibers in the walls of the leg veins B. contraction of smooth muscle in the walls of the leg veins C. contraction of skeletal muscles in your legs D. contraction of the left ventricle of the heart E. the valves in the leg veins alternating between open and closed 22. Elephantiasis is an infectious disease in which blockage of the circulatory system by microscopic, parasitic roundworms results in a dramatic swelling of the tissues in the lower half of the body. What region of the circulatory system is most likely blocked by these small, thread-like worms? A. lymphatic vessels and/or lymph nodes B. a carotid artery C. a pulmonary artery D. the superior vena cava E. the left atrium of the heart 23. What is true about the lymphatic system? A. Fluid moves through the lymphatic system under high pressure. B. It exhibits peristaltic contractions. C. It collects fluid that seeps into the tissues and returns it to the blood. D. It is bi-directional. E. It carries all absorbed materials from the stomach and intestines to the liver. 24. The cells that normally lack a nucleus and have a relatively short life span in humans are A. neutrophils. B. basophils. C. lymphocytes. D. erythrocytes. E. eosinophils. 25. Under what circumstance would your body experience an increase in erythropoietin secretion? A. if you are suffering from hypertension B. if you are suffering from dehydration C. after drinking 5 liters of water in just 2 hours D. if your hematocrit is elevated above normal levels E. if you just moved from New York City to start training for a marathon in the 'Mile-High' city of Denver, Colorado 26. What class of sensory receptors directly affects the activity of the cardiac center in the medulla oblongata? A. osmoreceptors in the hypothalamus B. blood glucose sensors in the pancreas C. sensory cells in the juxtaglomerular apparati of the kidneys D. blood pH chemoreceptors in the aortic and carotid bodies E. baroreceptors in the arch of the aorta and in the carotid arteries 27. Scientists believe that the heart of the common (chordate) ancestor of all vertebrates A. had four chambers. B. had two chambers. C. was little more than a heavily-muscled region of the ventral artery. D. had three chambers. E. consisted of a single ventricle. 28. When submerged, frogs receive most of their O2 from capillaries within the skin. What structure would contain blood with the highest O2 concentration in a submerged frog, which is not able to breathe using its lungs? A. right atrium B. left atrium C. ventricle D. pulmonary vein E. aorta 29. The left ventricle in mammals is more muscular than the right ventricle because A. the right ventricle only pumps blood into the right atrium. B. the left ventricle pumps blood with a higher hematocrit than the blood that leaves the right ventricle. C. the left ventricle is always contracting to maintain a systemic blood pressure. D. the left ventricle has to pump blood throughout the body at high pressure. E. the left ventricle pumps more total blood than the right ventricle. 30. What would happen if your SA node and AV node fired at same time? A. The QRS wave on an ECG would disappear, as it would be masked by the P wave. B. Blood would be forced back into the atria from the ventricles. C. The production of nitric oxide by your endothelial cells would increase. D. Your stroke volume would decrease. E. Your blood pressure would increase. 31. The blood pressure in your brachial artery is at its lowest during A. ventricular contraction. B. the diastolic period. C. the time the aorta is maximally stretched. D. exercise. E. the systolic period. 32. If the resting heart rate of an average individual is 72 beats per minute (bpm), what was the most-likely resting heart rate of swimmer Michael Phelps during the 2012 London Olympics? Note: Phelps has won more Olympic medals than any other athlete and was voted Associated Press Male Athlete of the Year in 2008 and 2012. A. 10 bpm B. 38 bpm C. 72 bpm D. 100 bpm E. 150 bpm 33. Which secreted molecule is LEAST likely to effect a significant change in blood pressure? A. aldosterone B. atrial natriuretic hormone C. high-density lipoprotein D. nitric oxide E. antidiuretic hormone 34. What animal group is mismatched with its circulation system type (open or closed)? A. lobster—open B. oyster—open C. killer whale—closed D. cricket—closed E. earthworm—closed 35. What is true about about mammalian red blood cells? A. Red blood cells contain albumin. B. Red blood cells are also called leukocytes. C. A mature red blood cell is shaped like a biconcave disc because it has no nucleus. D. Aged red blood cells are removed from the blood by the kidneys. E. Red blood cells develop from lymphoid stem cells. 36. In the blood clotting cascade mechanism, which specific enzyme catalyzes the conversion of fibrinogen to fibrin? A. renin B. catalase C. albumin D. thrombin E. angiotensin 37. One mechanism that some animals use to limit heat loss in cold environments is A. vasodilation of below surface blood vessels. B. increasing cardiac output. C. vasoconstriction of the precapillary sphincters in the skin. D. vasoconstriction of veins. E. vasodilation of the near surface blood vessels. 38. Edema is a result of increased interstitial fluid in various tissues of the body. All of the following are causes of edema EXCEPT A. protein malnutrition. B. liver disease. C. pregnancy. D. abnormally low blood pressure. E. injury to a limb. 39. The greatest metabolic advantage of a fish's two-chambered heart is A. its ability to stay full of blood even as it is emptying into the conus arteriosus. B. its ability to maintain a constant blood pressure in the fish. C. its ability to pump blood to the gills for gas exchange before the blood is pumped to the remaining areas of the body. D. its ability to keep oxygen-rich blood in one chamber separate from oxygen-poor blood in the other chamber. E. its ability to pump blood under high pressure to all areas of the fish's body. 40. When lungs evolved in vertebrates, the evolution of also occurred in the circulation of blood within those vertebrates. A. a lymphatic system B. a muscular heart C. red blood cells D. double circulation E. more plasma 41. The depolarization of the heart is initiated by the A. Purkinje fibers. B. left and right bundle branches. C. atrioventricular node. D. sinoatrial node. E. bundle of HIS. 42. A friend asks you to explain the difference between atherosclerosis and arteriosclerosis. You say, "Well A. atherosclerosis is a condition that results from having angina pectoris occurring in the arteries of the heart, while arteriosclerosis results from the accumulation of cholesterol deposits in the arteries." B. atherosclerosis is a condition that results from the accumulation of cholesterol deposits in the arteries, while arteriosclerosis results from too much calcium being deposited in the arteries." C. atherosclerosis is a condition that results from too much calcium being deposited in the arteries, while arteriosclerosis results from having angina pectoris occurring in the arteries of the heart." D. atherosclerosis is a condition that results from too much calcium being deposited in the arteries, while arteriosclerosis results from the accumulation of cholesterol deposits in the arteries." E. atherosclerosis and arteriosclerosis are nearly the same and usually occur after a stroke." 43. Which animal is likely to have the circulatory pattern shown in the figure? A. frog B. bird C. crocodile D. fish E. mammal 44. What factor most limits a person’s maximal increase in cardiac output during exercise? A. the time available to fill the heart chambers with blood as the heart rate increases B. the rate at which oxygen is delivered to the cardiac muscle cells C. the total amount of blood in an individual D. the time it takes for electrical impulses to reach all parts of the heart E. the rate at which cardiac muscle contracts 45. The loudest sounds a nurse hears while listening to a patient's heart through a stethoscope are caused by A. closing of the AV valves. B. depolarization of the ventricles. C. contraction of the atria. D. opening of the semilunar valves. E. contraction of the ventricles. 46. In a vertebrate with a four-chambered heart, the structure(s) containing blood with the highest O2 concentration is/are the . A. pulmonary arteries B. pulmonary veins C. right atrium D. inferior vena cava E. right ventricle 47. A nurse takes your blood pressure during your annual examination, and she enters a reading of 115/70 in your medical chart. These numbers mean that the A. nurse heard sounds through the stethoscope only when the pressure in your brachial artery was exactly 115 and again when it was exactly 70. B. pressure in your brachial artery during ventricular contraction is 115, while the pressure during ventricular relaxation is 70. C. diastolic pressure is 115 and the systolic pressure is 70. D. pressure in your brachial artery during ventricular contraction is 115 while the pressure during atrial contraction is 70. E. pressure in the ventricles of the heart is 115 when the ventricles are contracting and 70 when the ventricles are relaxing. 48. The P wave of the ECG represents A. the contraction of the atria. B. the depolarization of the atria. C. the contraction of the ventricle. D. the relaxation of the atria. E. the repolarization of the atria. 49. The figure below shows the blood pressure in the brachial artery for an average person, as it oscillates between the systolic pressure (120 mm Hg) and the diastolic pressure (75 mm Hg). Superimposed on the right side of the graph is a tracing of the pressure in the left ventricle, which drops to close to zero mm Hg during diastole. What prevents the blood pressure in the brachial artery from similarly dropping to almost zero during diastole? A. Forceful closing of the aortic semilunar valve propels the blood forward during diastole. B. Inhaling and exhaling propels the blood forward during diastole. C. Recoil of elastic fibers in the aorta propels the blood forward during diastole. D. Rhythmic contraction of smooth muscle tissue in the subclavian and brachial arteries propels the blood forward during diastole. E. Forceful closing of valves in the subclavian and brachial arteries propels the blood forward during diastole. 50. If your blood has a higher than normal concentration of albumin, what is also likely to be true? A. The osmotic pressure of your interstitial fluid is higher than normal. B. Six hours previously, you ate a very large steak, which was rich in muscle proteins. C. The net reabsorption of interstitial fluid in the capillaries is higher than normal. D. Proteins are escaping into the interstitial fluid, causing a reduction in blood volume. E. Your hematocrit is abnormally high. 51. Stimulating the vagus nerve would cause A. increased firing of the SA and AV nodes B. heart rate to decrease. C. blood pressure to rise. D. a release of epinephrine. E. blood volume to increase. 52. Myeloid stem cells give rise to all formed elements in the blood EXCEPT A. erythrocytes. B. platelets. C. monocytes. D. lymphocytes. E. basophils. 53. What type of mollusk has a closed circulatory system? A. squid B. snail C. clam D. scallop E. slug Cephalopods are active predators with large brains to coordinate complex behaviors that require lots of nerve and muscle activity. A closed circulatory system enables them to meet these high metabolic demands. 54. You are taking a lab practical exam, and one station has pins in various internal structures of a bisected heart. You can orient yourself without picking up and examining the exterior of the heart because A. the width of the external wall of the right ventricle is thicker than all the other chamber walls. B. the width of the external wall of the right atrium is thicker than all the other chamber walls. C. the width of the external wall of the left atrium is thicker than all the other chamber walls. D. the ascending aorta exits from the right ventricle. E. the width of the external wall of the left ventricle is thicker than all the other chamber walls. 55. If an arteriole has a radius of 48 micrometers (m) and a capillary in the same organ has a radius of 8 m, what is the difference in resistance in blood flow between the two? A. The resistance in the arteriole is 6 times the resistance in the capillary. B. The resistance in the capillary is 6 times the resistance in the arteriole. C. The resistance in the arteriole is 48 times the resistance in the capillary. D. The resistance in the capillary is 48 times the resistance in the arteriole. E. The resistance in the arteriole is 96 times the resistance in the capillary. F. The resistance in the capillary is 96 times the resistance in the arteriole. 56. Animals with closed circulatory systems are more successful than animals with open circulatory systems. FALSE FALSE!! All animals living today are equally successful, since they have been evolving for the same amount of time from their shared common ancestor. Simple doesn’t mean unsuccessful! Simpler organisms are well adapted to do what they do, but they do less than more complicated organisms. However, animals with closed circulatory systems usually have higher levels of metabolic activity (e.g., greater need for oxygen and sugar to make lots of ATP) than animals with open systems. 57. Susie just squished a cockroach and its pale yellow hemolymph squirted out on the floor. If Susie analyzed the hemolymph, she would expect to find all of the following EXCEPT A. water. B. erythrocytes. C. ions. D. cells that protect against foreign microbes. E. amino acids. F. sugars. Insects do not have red blood cells (or respiratory pigments), as they use their tracheal system (and not their open circulatory system) to transport oxygen and carbon dioxide directly to and from cells, respectively. The absence of respiratory pigments results in the pale yellow color of the hemolymph. Chapter 48: The Respiratory System 1. Most simple invertebrates obtain their oxygen by diffusion directly from the surrounding A. mud. B. water. C. blood. D. air. 2. Fish and crustaceans are similar in that both use for gas exchange. A. tracheal tubes B. lungs C. the moist surface of their skin D. gills E. the walls of the mantle cavity 3. The large number of alveoli in mammalian lungs, the numerous lamellae in fish gills, the papulae of echinoderms, and the extensive tracheal system of insects are all adaptations that optimize what factor in the formula for Fick’s Law of diffusion? A. p B. A C. d D. D 4. Which step in the respiratory process of mammals requires the most energy? A. exhalation B. transport of oxygen across the walls of the alveoli C. inhalation D. binding of oxygen to hemoglobin E. transport of carbon dioxide across the walls of the alveoli F. activity of carbonic anhydrase in the lungs 5. Each breath is initiated by neurons in the "respiratory control center" located in the A. spinal cord. B. cerebellum. C. larynx. D. carotid body. E. medulla oblongata. 6. In tissue capillaries, combines with water to form carbonic acid, which after being transported to the lungs, dissociates back to its constituents. A. CO2 B. O2 C. H2CO3 D. hemoglobin E. HCO - 7. Chemoreceptors located in the aortic and carotid bodies stimulate the respiratory control center in the brain when the blood decreases. A. pH B. PO2 C. PCO2 D. hemoglobin concentration E. HCO - concentration 8. Ciliated gills in clams, sharks continuously swimming with their mouths partly open, countercurrent flow of blood and water in fish gills, and the use of air sacs to effect unidirectional flow of air through bird lungs are all adaptations that optimize what factor in the formula for Fick’s Law of diffusion? A. D B. p C. d D. A 9. Imagine that you have been transported light years away to another planet with water-filled oceans. This distant planet has a barometric pressure of 1,000 mm Hg at sea level and air that contains 15 percent O 2. Would you take in much more oxygen, much less oxygen, or about the same amount of oxygen per breath on this planet as you would on Earth? A. much more oxygen on the distant planet B. much less oxygen on the distant planet C. about the same amount of oxygen as on Earth 10. The general term that describes the uptake of oxygen from the environment and the disposal of carbon dioxide at the body system level is A. inspiration. B. diffusion. C. respiration. D. cellular respiration. E. exhalation. 11. The exchange of gases across a respiratory surface always occurs by A. bulk flow. B. facilitated transport. C. diffusion. D. active transport. E. osmosis. 12. Tim, who is an avid cyclist, is struck by a car during an organized bike ride. Two of his ribs are broken, puncturing his chest wall on the right side. What will most likely happen to his lungs? A. His right lung will overinflate and his left lung will collapse. B. Both lungs will collapse. C. His left lung will be okay, but his right lung will collapse. D. Both lungs will overinflate. E. His right lung will collapse and his left lung will overinflate. F. His left lung will be okay, but his right lung will overinflate. 13. The most efficient lungs with respect to oxygen uptake are found in this group of vertebrates. A. amphibians B. birds C. fish D. mammals E. reptiles 14. The oxygen content of dry air in the atmosphere A. is about 78.1%. B. is about 0.03%. C. varies depending on the altitude. D. is about 21.0%. E. is inversely proportional to the partial pressure of oxygen. 15. Fick's law of diffusion states that the rate of diffusion of a gas across a membrane depends on all of the following except A. the width of the membrane. B. which membrane transport proteins are present. C. the size of the gas molecule. D. the membrane surface area. E. the concentration difference of the gas on the two sides of the membrane. 16. The external gills of an adult axolotl are not as efficient as fish gills because . A. blood flows though the axolotl gills more slowly than through fish gills B. water flows over the axolotl gills at many different angles C. axolotl gills are not highly branched D. axolotl gills are covered by thick layers of cells E. axolotls are slow swimmers and all fish swim fast to ventilate their gills 17. The principal reason(s) for why gills were replaced with different types of respiratory organs in terrestrial animals is (are) I-tissues and organs are less buoyant in air than in water. II-countercurrent exchange is not very efficient at maximizing blood oxygenation levels. III-water diffuses into air through evaporation. A. just I B. I and III C. I, II, and III D. just II E. II and III 18. Where a given species of vertebrate lives has dramatically shaped the evolution of respiratory adaptations, particularly since the oxygen concentration in air is approximately 20 times that of the oxygen concentration in water. Per unit area of gas exchange surface, which group of vertebrates can extract the greatest total amount of oxygen from the external medium? A. fish B. mammals C. reptiles D. amphibians E. birds Amphibian lungs are not as as highly-branched as reptile, bird or mammal lungs. Amphibians supplement gas exchange in their small lungs by using cutaneous respiration and, in some cases, gills. 19. Four-year old Sammy is having a temper-tantrum. In a final attempt to get his way, he yells to his mom, “I'm gonna hold my breath ‘til I die!” What is the primary reason that Sammy’s mom is unconcerned by this statement? A. It is impossible for Sammy to control the contraction of his diaphragm and intercostal muscles since they are made up of smooth muscle. B. As Sammy holds his breath, the increasing levels of carbon dioxide in his blood will lead to stimulation of the respiratory control center in his brain. C. Sammy is too young to hold his breath for more than 30 seconds. D. As Sammy holds his breath, the decreasing levels of oxygen in his blood will lead to stimulation of the respiratory control center in his brain. 20. When is the PO2 an especially strong stimulant of the respiratory control center in the brain of healthy individuals? A. while hyperventilating B. while free-diving to a coral reef 60 feet below the surface of the sea C. while climbing to the summit of Mt. Everest D. while hypoventilating E. while running a road race along the shoreline of Vancouver 21. Which blood vessels carry oxygenated blood? A. pulmonary veins B. systemic veins (such as the femoral vein in your leg) C. pulmonary arteries 22. What is the likely outcome if you were able to force water through fish gills in the opposite direction to how it normally flows over the gills? A. The blood's oxygen concentration would not be able to exceed 50%. B. The direction of blood flow would reverse through the gills to compensate for the reversed direction of water flow. C. There would be no effect, and maximal oxygenation of blood would still occur. D. The movement of carbon dioxide from the blood into the water would increase. E. Gas exchange would be inhibited and movement of oxygen into the blood would cease. 23. What change would increase the rate of oxygen diffusing from lung alveoli into the blood? A. A decrease in the partial pressure of oxygen in the alveoli B. An increase in the partial pressure of oxygen in the blood entering the lungs C. An increase in the humidity of air in the lungs D. A decrease in the pH of the blood entering the lungs E. An increase in the hemoglobin concentration of the red blood cells 24. One possible reason that there are not any really large insects currently living on Earth is that A. their gills do not have as many lamellae as fish gills. B. gas exchange across their moist skin surfaces cannot support the metabolic needs of a large body. C. the bulk flow of air through their tracheal tubes would be more difficult the larger the body and longer the tracheal tubes. D. their lungs never evolved to be as highly branched as a mammal's. E. they do not have as many papulae as echinoderms. 25. During your dissection of a fish, what structure would you have to remove to see the gills? A. trachea B. jaw bone C. spiracle D. glottis E. operculum 26. In the countercurrent exchange in a fish's gills, the oxygen concentration is always in the water than the blood along the entire pathway of each lamella. A. higher B. lower C. the same 27. Which of these animals uses positive pressure breathing? A. mammals B. birds C. amphibians D. reptiles 28. Which group of animals has one-way flow of air through its lungs, whereas all the others have tidal flow? A. amphibians B. reptiles C. birds D. mammals 29. During an asthma attack, allergens trigger the release of histamine, which causes severe constriction of the . A. larynx B. pleural cavities C. trachea D. bronchi E. rib cage 30. The gas exchange surfaces in a mammalian lung are organized into structures that look like clusters of hollow grapes. The hollow, grape-like structures are the . A. papulae B. bronchi C. lamellae D. alveoli E. spiracles 31. The respiratory systems of fish and birds are similar in that both . A. utilize air sacs. B. have similar partial pressures of oxygen in the environmental medium. C. employ countercurrent exchange. D. have unidirectional flow of the environmental medium over the gas exchange surfaces. 32. A major genetic cause of emphysema is deficiency of the protein alpha-1 anti-trypsin (AAT). AAT is an enzyme that inhibits the activity of another protein. What protein is most likely the normally-inhibited target of AAT? A. hemocyanin B. hemoglobin C. elastin-specific protease D. myoglobin E. carbonic anhydrase F. collagen 33. In the future, it may be possible to screen for a number of different types of cancer by a simple blood test. If a blood test indicates a very early stage of lung cancer, where is the most likely place that the tumor will be growing? A. in the parietal pleural membrane B. in the visceral pleural membrane C. in the wall of an alveolus D. in the wall of a large bronchus E. in the wall of a pulmonary vein 34. Tom is choking on a piece of steak that is stuck in his trachea, and he signals for his young son to call 911. It takes the paramedics four minutes to reach his house. Will this be in time to save Tom (and why), or will Tom already be dead (and why)? A. Tom will be dead due to the lowered pH of his blood. B. Tom will be dead since all the hemoglobin in his arterial blood will unload its oxygen during the first pass through the tissues after his last breath. C. Tom will be alive since the myoglobin in his muscles will release oxygen into the blood after he stops breathing. D. Tom will be alive since he had a reserve of about three-quarters of his hemoglobin still bound to oxygen when he started choking. 35. Which dissolved gas cannot be transported by hemoglobin? A. carbon dioxide (CO2) B. carbon monoxide (CO) C. nitric acid (NO) D. nitrogen (N2) E. oxygen (O2) 36. The primary advantage of gills versus the body surface for gas exchange in aquatic animals is that . A. gills decrease resistance to movement B. all gills contain a countercurrent exchange system C. gills have a much greater diffusion surface area D. gills are located near the mouth 37. What respiratory adaptation was important for the invasion of land by arthropods? A. the ability to close spiracles to prevent water loss B. cutaneous respiration in moist habitats C. countercurrent exchange surrounding the trachea D. gas- and water-permeable exoskeletons E. positive pressure breathing to increase gas exchang 38. Humans cannot survive long at altitudes greater than 6000 m because . A. there is much more CO2 in the atmosphere at the higher altitudes than at sea level B. the diaphragm cannot be contracted enough to produce a full breath C. the partial pressure of O2 becomes less than half that at sea level D. alveoli become compressed at the lower atmospheric pressures 39. Soft-shelled turtles can remain submerged underwater for several hours without ventilating their lungs because . A. the partial pressure of O2 is higher underwater B. they also have rudimentary gills C. they can store some air in special air sacs under the shell D. they can use cutaneous respiration through the soft lining of their throats 40. Major evolutionary changes have resulted in anatomical and physiological adaptations that optimize the rate of gas exchange in animals by each of these factors except . A. decreasing the distance for diffusion B. increasing the surface area for diffusion C. increasing the diffusion constant for each gas D. increasing the concentration difference driving diffusion 41. Most phyla of aquatic invertebrates have improved the rate of gas exchange by creating a water current over their respiratory surfaces, thereby . A. decreasing the distance for diffusion B. increasing the surface area for diffusion C. changing the diffusion constant D. increasing the concentration difference 42. The large number of alveoli in lungs improve gas exchange by . A. decreasing the distance for diffusion B. increasing the concentration difference C. changing the diffusion constant D. increasing the surface area for diffusion 43. The many lamellae in the gills of fish improve respiratory efficiency by . A. changing the diffusion constant B. increasing the surface area for diffusion C. increasing the concentration difference D. decreasing the distance for diffusion 44. The close proximity of capillaries to the alveoli in lungs improve respiratory efficiency by . A. increasing the surface area for diffusion B. decreasing the distance for diffusion C. changing the diffusion constant D. increasing the concentration difference 45. The North American pronghorn antelope can run almost as fast as a cheetah, but unlike the cheetah, can also sustain speeds of 30-40 mph for miles. The pronghorn is about the size of a large, domestic goat. You would predict the antelope to have a than the goat. A. smaller trachea B. smaller lung volume C. higher concentration of hemoglobin D. lower concentration of myoglobin E. lower concentration of bicarbonate 46. If each human lung contains about 300 million alveoli with a summed total surface area for gas exchange of 40 square meters, what is the approximate surface area of a single alveolus? A. 0.133 square millimeters B. 133 square millimeters C. 1.33 square micrometers D. 7.5 square millimeters E. 7.5 square micrometers F. 750 square micrometers 1 meter (m) = 1,000 millimeters (mm). Thus, 1 m2 = 1 x 106 mm2 and 40 1 x 106 mm2 300 x 106 alveoli = 0.133 mm2/alveolus 47. What is the order of structures or spaces that a surgeon will pass through to operate on a human lung? A. skin -> intercostal muscles -> parietal pleural membrane -> pleural cavity -> visceral pleural membrane -> lung tissue B. skin -> pleural cavity -> intercostal muscles -> visceral pleural membrane -> parietal pleural membrane -> lung tissue C. skin -> intercostal muscles -> pleural cavity -> parietal pleural membrane -> visceral pleural membrane -> lung tissue D. skin -> visceral pleural membrane -> intercostal muscles -> pleural cavity -> parietal pleural membrane -> lung tissue E. skin -> intercostal muscles -> visceral pleural membrane -> pleural cavity -> parietal pleural membrane -> lung tissue 48. During exhalation, the diaphragm . A. relaxes and moves up B. relaxes and moves down C. contracts and moves down D. contracts and moves up 49. Which part of a hemoglobin molecule binds to a molecule of O2? A. the surface of an globin chain B. the surface of a globin chain C. the hydrophobic pocket between two chains D. the copper atom wedged between an and a globin chain E. the iron atom in the center of a heme group 50. In which PO2 environment would hemoglobin most easily release its O2? A. 65 mm Hg B. 40 mm Hg C. 100 mm Hg D. 75 mm Hg E. 90 mm Hg 51. During exercise, the blood returning to a person's heart contains hemoglobin that is about saturated. A. 3% B. 22% C. 35% D. 75% E. 97% 52. Every winter, many individuals tragically die in their sleep due to faulty heaters that leak carbon monoxide (CO) during the night. How does CO poisoning lead to death? A. CO binds more tightly to hemoglobin than does oxygen. B. CO binds more tightly to carbonic anhydrase than does carbon dioxide. C. CO inhibits the chemosensitive neurons in the aortic and carotid bodies. D. CO coats the surfaces of the alveoli, preventing diffusion of oxygen into the blood. E. CO binds to oxygen, and the CO-oxygen complex cannot bind to hemoglobin. 53. Protruding papulae on the surface of echinoderms improve respiratory efficiency by . A. changing the diffusion constant B. decreasing the distance for diffusion C. increasing the concentration difference D. increasing the surface area for diffusion 54. The axolotl, an aquatic amphibian, uses for gas exchange. A. external gills B. lungs C. papulae D. internal gills E. tracheal tubes 55. Ram ventilation is used by during respiration. A. penguins B. whales C. frogs D. tuna E. salamanders 56. Air enters human lungs from the mouth by passing through the . A. trachea B. esophagus C. spiracles D. nasal cavities E. posterior air sacs Chapter 45: The Endocrine System 1. Your general practitioner considers your unusual set of symptoms, and says, "You seem to have a problem with maintaining homeostasis in your body." Which type of specialist might the doctor refer you to? (Check two.) hematologist X endocrinologist X neurologist osteologist opthamologist obstetrician-gynecologist 2. A is a regulatory molecule that is secreted into the blood by an endocrine gland. A. metabolite B. hormone C. peptide D. G protein E. neurotransmitter 3. A hormone is a regulatory molecule that is secreted into the blood by an endocrine gland. What is wrong with this definition? A. Endocrine glands are the target of these molecules, not the source. B. It does not apply to this type of molecule in plants. C. The molecules do not serve a regulatory role. D. The definition is completely correct as stated. 4. You are a professor supervising first year medical students. One student is examining the adrenal gland of his cadaver. "Wow!" he exclaims, "This person must have had an endocrine problem. Their adrenal gland lacks any kind of duct connecting it to the bloodstream. The hormones could only have oozed out." How do you respond? A. "That makes their adrenal gland similar to an exocrine organ." B. "Do you see signs of adrenal insufficiency, such as low fat stores or muscle degeneration?" C. "That's normal. The hormones ooze out and diffuse into nearby capillaries, which is why endocrine glands are sometimes called 'ductless glands'." D. "Look carefully and you will see the nerves that transport the hormones into the bloodstream." 5. Some intercellular regulatory molecules that exert only very local effects are called regulators. A. sympathetic B. exogeneous C. mesocrine D. endocrine E. paracrine Sometimes the most practical way to do an experiment is not to perform it in an animal, but to look at cell responses in culture. Cells from mice, humans, and other mammals have been used to establish cell culture lines that have been very important for research. You are studying a novel water-soluble mouse hormone. You know cell culture can be a practical model to reveal protein function, so you apply the hormone to yeast cells, but nothing happens. 6. What is a likely explanation for why nothing happened in your experiment? A. Yeast have a cell wall, so the molecule cannot pass through the cell membrane as it would in mice. B. Water-soluble hormones are lipids, not proteins. C. You need to apply the hormone in large amounts to see an effect. D. Yeast may lack the receptor required for the hormone to act. 7. How could you improve your experiment? A. Apply the hormone to mouse cell culture. B. Apply the hormone at a concentration of 0.5M. C. Hydrolyze the hormone before application. D. Add ATP as a cofactor. 8. The lobe of the pituitary gland appears fibrous because it contains axons originating from hypothalamic neurons. A. anterior B. posterior C. superior D. inferior E. medial 9. The anterior pituitary gland is controlled by which three types of signals? (Check three.) direct axonal connections from the hypothalamus X negative feedback from its own hormones X negative feedback from hormones secreted by the thyroid, adrenal cortex, and gonads positive feedback from its own hormones positive feedback from hormones secreted by the pancreas, heart, and skin X releasing hormones secreted by the hypothalamus 10. There is a molecule in black licorice that inhibits an enzyme which normally limits activation of mineralocorticoid receptors. Thus, overconsumption of licorice can mimic the effect of excess aldosterone. What would be the result? A. infertility in men B. hypertension (high blood pressure) due to insufficient potassium in the blood C. excessive body hair D. low blood pressure due to excessive potassium in the blood 11. A novel disease causes destruction of the adrenal glands. If a patient with this disease failed to receive aldosterone replacement therapy, what would result? A. death B. diminished cognitive function C. infertility in both men and women D. poor blood sugar regulation 12. The islets of Langerhans located in the secrete both insulin and glucagon. A. liver B. pituitary C. thalamus D. kidney E. pancreas 13. Ecdysone and juvenile hormone regulate molting and in insects. A. metamorphosis B. sexual behavior C. body shape D. egg production 14. Oxytocin is a peptide hormone that causes uterine contractions in childbirth and stimulates milk secretion from nipples. Do men have oxytocin? YES In addition to the functions associated with childbirth and nursing, oxytocin is sometimes called the "cuddle hormone" for its roles in pair bonding and sexual arousal, which it promotes in both sexes. 15. Women often receive synthetic oxytocin to induce childbirth or augment contractions. What is an important consideration about the use of synthetic oxytocin? A. It is impossible to mimic its chemical structure, so it may act on other receptors and cause side effects. B. The synthetic form does not travel down motor neurons, so it is less effective than the natural form. C. It can be difficult to get the levels just right, so uterine contractions can be too strong. D. If you fail to simultaneously provide synthetic G protein, the hormone is ineffective. 16. A couple visits a fertility clinic to determine the basis of their difficulty in conceiving a child. They are surprised to hear they will both be tested for levels of two of the same hormones, which play roles in the ovarian cycle and sperm production. Which hormones are they? A. adrenocorticotropic hormone (ACTH) and corticotropin-releasing hormone (CRH) B. insulin and glucagon C. oxytocin and luteinizing hormone (LH) D. follicle-stimulating hormone (FSH) and luteinizing hormone (LH) 17. The use of bovine growth hormone (BGH) to increase milk yield in dairy cows is highly controversial. What other hormone is regularly added to milk, yet causes little controversy? A. vitamin A B. vitamin D C. MSG D. preservatives After addition of two hydroxy groups in the body, Vitamin D becomes a hormone that promotes absorption of Ca+2 by the intestines, which improves bone mineralization and prevents rickets. 18. What is the primary function of the hormone ADH? A. regulates biological rhythms B. mediates adaptation to long term stress C. stimulates bone breakdown D. conserves water by stimulating its reabsorption from urine E. inhibits loss of calcium from bone 19. What is a primary function of growth hormone? A. initiates stress response B. raises blood glucose level C. lowers blood glucose level D. Stimulates bone growth E. inhibits loss of calcium from bone 20. What is a primary function of the hormone calcitonin? A. initiates stress response B. raises blood glucose level C. lowers blood glucose level D. stimulates bone growth E. inhibits loss of calcium from bone 21. Lampreys completely lack parathyroid glands. Suggest why these glands may be important for other vertebrates, but not for jawless fish. A. Because they are parasitic, lampreys get plenty of calcium from their hosts. B. Living in the ocean, they are surrounded by large amounts of sodium, chloride, and magnesium ions. C. Jawless fish employ a different system than other vertebrates to regulate blood sugar. D. Since they have a cartilaginous skeleton but not true bone, they have less need to regulate calcium levels. 22. Which hormone helps with adaptation to long term stress? A. glucocorticoids B. ephinephrine C. insulin D. glucagon E. melatonin 23. Which hormone lowers blood glucose levels? A. glucocorticoids B. ephinephrine C. insulin D. glucagon E. melatonin 24. Which hormone raises blood glucose levels? A. glucocorticoids B. ephinephrine C. insulin D. glucagon E. melatonin 25. At the breakfast table, your roommate says, "Oops! I forgot to take my melatonin last night. I'd better take it now." How do you respond? A. "Good idea -- you don't want your bones to get brittle." B. "Be sure not to take too much, or your blood sugar could get dangerously low." C. "Don't take it now - it regulates biological rhythms, and is normally high at night and low during the day." D. "Don't take it after a meal - it promotes the 'fight-or-flight' response, not 'rest-and-digest'." 26. Which hormone sets the body's basal metabolic rate? A. estrogen B. insulin C. norepinephrine D. cortisol E. thyroxin 27. The hormones epinephrine and norepinephrine constitute the "alarm" response of the body to stress. What secretes these hormones? A. pancreas B. thyroid gland C. adrenal medulla D. pineal gland E. anterior pituitary gland 28. What is an example of a molecule that can act as both a neurotransmitter and a hormone? A. estrogen B. progesterone C. thyroxine D. norepinephrine E. insulin 29. Which of the following is a paracrine regulator that affects almost every organ in the body? A. insulin B. testosterone C. prostaglandin D. gonadotropin E. epinephrine 30. Which of the following hormones exerts its action by interacting with a receptor at the cell surface? A. insulin B. cortisol C. testosterone D. estrogen E. progesterone 31. Which hormone is NOT produced by the anterior pituitary gland? A. prolactin B. lutenizing hormone C. calcitonin D. growth hormone 32. Why does consumption of alcohol stimulate urination? A. It stimulates ADH secretion. B. It inhibits ADH secretion. C. It stimulates vasopressin. D. It stimulates oxytocin. E. It inhibits aldosterone. 33. What hormone stimulates the milk-ejection reflex? A. antidiuretic hormone B. oxytocin C. vasopressin D. parathyroid hormone E. aldosterone 34. Hormones of the posterior pituitary gland are transported to the hypothalamus by what unusual route? A. the limbic system B. blood vessels C. axons D. the lymphatic system E. the hypopituitary shunt 35. What hormone stimulates milk production in mammals? A. aldosterone B. melatonin C. prolactin D. luteinizing hormone E. growth hormone 36. Which of the following are examples of gonadotropins? A. prolactin and oxytocin B. follicle-stimulating hormone and oxytocin C. luteinizing hormone and estrogen D. prolactin and estrogen E. follicle-stimulating hormone and luteinizing hormone 37. Nonpolar hormones can pass through the intestinal cell membrane without being digested, and therefore can be taken orally. Which of the following hormones are available as pills? A. insulin and growth hormone B. thyroid hormone and steroid hormones C. calcitonin and insulin D. insulin and steroid hormones E. calcitonin and thyroid hormone 38. Which is NOT an effect of the hormonal products of the adrenal medulla? A. accelerate heartbeat B. stimulate synthesis of glycogen C. increase blood pressure D. reduce blood flow to the skin and digestive organs E. increase blood sugar 39. Which one of the following is a glucocorticoid important in both gluconeogenesis (glucose produced from amino acids) during starvation and anti-inflammation? A. aldosterone B. cortisol C. glucagon D. insulin E. prolactin 40. Which of the following hormones is NOT produced by the adrenal gland? A. epinephrine B. cortisol C. prolactin D. aldosterone E. norepinephrine 41. Which two hormones interact to maintain normal levels of blood glucose? A. cortisone and testosterone B. epinephrine and norepinephrine C. glucagon and insulin D. LH and FSH E. oxytocin and vasopressin 42. When the body's blood glucose level rises above normal, which of the following hormones is produced? A. glucagon B. epinephrine C. insulin D. norepinephrine E. prolactin 43. Which of the following conditions is not associated with diabetes? A. elevated levels of blood glucose B. lowered production of insulin by the pancreas C. reduced body cell sensitivity to insulin D. defects in pituitary gland function E. careful diet and exercise measures are required to balance blood sugar levels 44. The right atrium secretes atrial natriuretic hormone (ANH), which stimulates the kidneys to excrete salt and water into the urine. ANH acts antagonistically to what other hormone? A. aldosterone B. cortisol C. glucagon D. thyroxine E. progesterone 45. Which of the following is NOT an example of a sex steroid hormone? A. estrogen B. progesterone C. prolactin D. testosterone E. estradiol 46. Which of the following hormones regulates metamorphosis and molting in insects? A. melatonin B. ecdysone C. estrogen D. somatostatin E. trypsin 47. Which two systems in the human body are devoted exclusively to the regulation of organ function? A. endocrine and circulatory systems B. respiratory and circulatory systems C. nervous and circulatory systems D. nervous and endocrine systems E. respiratory and nervous systems 48. Choose the hormone that is NOT correctly matched with its chemical category. A. polypeptides—ADH B. steroid—estradiol C. amine—FSH D. glycoprotein—LH E. steroid—testosterone 49. Which statement below does NOT correctly describe paracrine regulation? A. Paracrine regulatory molecules include cytokines and growth factors. B. Nitric oxide can function either as a neurotransmitter or a paracrine regulatory molecule. C. Endothelin and bradykinin are paracrine molecules that control blood vessel constriction and dilation. D. Paracrine molecules are released into the circulatory system to affect distant tissues. E. Platelet-derived growth factor and insulin-like growth factor are both paracrine regulatory molecules. 50. Choose the hormone that is NOT correctly matched with the endocrine gland that secretes it. A. adrenal cortex—aldosterone B. pancreas—insulin C. pineal gland—melatonin D. adrenal medulla—cortisol E. posterior lobe of pituitary gland—oxytocin 51. The cells of the anterior lobe of the pituitary gland secrete many hormones. From the following list select the hormone that is NOT secreted from the anterior lobe of the pituitary. A. growth hormone B. prolactin C. follicle-stimulating hormone D. melanocyte-stimulating hormone E. antidiuretic hormone 52. Large or polar hormones cannot enter a cell directly. Which of the following molecules can act as a second messenger for such hormones? A. catecholamine B. cyclic AMP C. epinephrine D. norepinephrine E. calcitonin 53. Which statement does NOT correctly describe hormones that can directly enter a cell? A. Steroid hormones can enter directly into cells because of their hydrophobic makeup. B. Water-soluble hormones can enter directly into cells because of their nonpolar makeup. C. Steroid hormones dissolve poorly in the plasma and are transported to target cells via special protein carriers. D. Once these hormones enter a cell, they bind to a cytoplasmic receptor and travel as a complex to the nucleus, or they may bind to the receptor after entering the nucleus. 54. Which of the following does NOT correctly describe antidiuretic hormone? A. It regulates water turnover in the mammalian kidney. B. ADH secretion is stimulated by high blood osmolarity. C. It ultimately acts to reabsorb more water. D. It exerts it effects via the cAMP pathway. E. It does not involve the use of aquaporin molecules. 55. Which hormone is secreted by alpha cells of the pancreas in response to hypoglycemia? A. glucagon B. insulin C. glucose D. cortisol E. glycogen 56. Which hormone is most similar in amino acid structure to chicken vasotocin? A. antidiuretic hormone B. aldosterone C. angiotensin II D. growth hormone E. oxytocin 57. Which is the best explanation for how ibuprofen works? A. It thins the blood to relieve joint pain. B. It inhibits cyclooxygenase-2, which is needed to produce prostaglandins. C. It activates cyclooxygenase-2, which then produces more prostaglandins. D. It down-regulates the production of substance P. E. It activates cyclooxygenase-2, which promotes healing of tissue. 58. Which of the following is likely to occur after drinking three alcoholic beverages? A. Increased urination due to alcohol activating antiduretic hormone production. B. Decreased urination due to alcohol activating aldosterone production. C. Increased urination due to alcohol inhibiting antidiuretic hormone production. D. Decreased urination due to alcohol inhibiting antidiuretic hormone production. E. Increased urination due to alcohol inhibiting aldosterone production. 59. Which of the following is NOT a function of oxytocin? A. pair bonding B. milk-ejection reflex C. milk production D. uterine contractions 60. Which of the following hormones has the fewest targets? A. growth hormone B. oxytocin C. melatonin D. thyroid-stimulating hormone E. testosterone 61. Which of the following is NOT a function of parathyroid hormone? A. It increases calcium production in the liver. B. It increases calcium absorption across the intestine. C. It increases reabsorption of Ca2+ in the kidney. D. It stimulates osteoclasts to dissolve CaPO4 crystals in bone. 62. Which of the following features distinguishes all hormones from neurotransmitters? A. Hormones require large concentrations to have an effect, while neurotransmitters require only low concentrations. B. A hormone affects reproductive functions, while neurotransmitters control all other processes. C. Hormones travel via the blood to a target tissue, while neurotransmitters cross the synaptic cleft to a postsynaptic cell. D. Hormones are steroids, while neurotransmitters are amino acids. 63. What is the difference between the endocrine system and the autonomic nervous system? A. The endocrine system employs chemical signals only, while the autonomic nervous system involves a combination of electrical and chemical signals. B. The autonomic nervous system promotes "fight or flight" functions, while the endocrine system promotes "rest and digest" functions. C. Norepinephrine is produced by the autonomic nervous system, while epinephrine is produced by the endocrine system. D. The axons of neurons of the endocrine system travel a shorter distance than those of the autonomic nervous system. 64. If blood transports hormones throughout the body, how do they communicate with specific targets? A. Special gateway valves in the blood vessels direct hormones to their target tissues. B. Special carrier proteins "walk" along microtubule tracts to deliver the hormones to their targets. C. Axonal pathfinding mediates the delivery of hormones to their specific targets. D. Only target tissues have receptors that allow them to receive the signal. 65. True or False: Hormones must be present at high concentrations to elicit an effect. FALSE Hormone concentrations in the blood can be as low as 10-10 M. 66. True or False: A molecule can act as both a neurotransmitter and a hormone. TRUE Examples of molecules that can act as both are epinephrine and norepinephrine. 67. True or False: The brain controls organ system function only through direct contact with sympathetic or parasympathetic nerves. FALSE Brain regions such as the hypothalamus, posterior pituitary, and pineal gland are endocrine organs that coordinate body functions via blood-transported hormones. 68. Which are endocrine organs? (Check all that apply.) X adenohypophysis X adrenal gland cochlea X hypothalamus ileum lymph node X neurohypophysis X ovary X pancreas X pineal gland salivary gland sebaceous gland spleen X testis X thyroid gland 69. Why don't hormones build up in the body over time? A. They are secreted from the body by exocrine glands. B. They are filtered out of the blood by lymph tissues. C. They do, but target tissues become desensitized over time. D. They are degraded by enzymes in the target tissue. 70. You are studying a disease caused by insufficient levels of a specific peptide hormone, and characterize mutations in several genes that can lead to the disease. Your startup company can only afford to pursue one gene product as a possible therapy. Which would you choose? A. Gene A -- a transcription factor. B. Gene B -- an enzyme that activates the hormone by cleaving its inactive precursor. C. Gene C -- the hormone itself. D. Gene D -- a kinase that acts downstream of the hormone. Peptide hormones are an attractive drug target since they are easily synthesized and become bioavailable as soon as they enter the bloodstream. Transcription factors are generally difficult to target with drugs. The enzyme is likely to be large and intracellular. The kinase is intracellular and likely functions in many other tissues, so targeting it may produce side-effects. The basic categories of hormones are peptide or proteins, steroids, and amino acid derivatives. The chemical properties of each hormone type determine some of its functional properties and how it must interact with receptors to signal to the cell. 71. A new hormone is discovered. Tests reveal that it is small, hydrophilic, and degraded by a protease. What type of hormone is it? A. peptide B. amino acid derivative C. steroid D. glucocorticoid E. prostaglandin 72. Is it likely that a small, hydrophilic hormone can cross the cell membrane? NO 73. What are the three main categories of hormones? (Check three.) adrenergic X amino acid derivative anabolic catabolic cytosolic X peptide / protein X steroid 74. You are researching an "orphan disease" with no available treatment. Your lab develops a drug that mimics a lipophilic hormone thought to be lacking in the disease, and you are looking for a pharmaceutical company to partner with to run a clinical trial. Based on the reaction from each team, which would you choose to work with? A. Company A: "What you've developed is wonderful, but we'll need to develop a delivery system to get the drug across the cell membrane." B. Company B: "Your new drug seems very promising. Do you know which G-protein coupled receptor it binds to?" C. Company C: "We're interested in developing this drug - it could really help people. Do you know which genes the hormone regulates?" 75. You are an attending physician supervising a new resident. Together, you decide to treat a patient with a steroid drug. The resident administers the drug and watches the patient anxiously. "It's not working," he mutters. How do you respond? A. "You're right, it doesn't seem to be working. Let's increase the dose, since we need to achieve high steroid levels in the blood." B. "Why don't you go on your rounds and come back? Steroid hormones need several hours to work, since they regulate gene expression. Transcription and translation take time." C. "We forgot to administer the carrier protein to get the drug across the cell membrane. We can add that now." D. "Let's grab a cup of coffee and then check again. Activating the G-protein coupled receptor and its downstream kinase cascade can take several minutes." 76. You are scheduled to give a seminar to your fellow med school students, and have chosen to speak about the pituitary. What important points will you include? (Check all that apply.) X The posterior pituitary is derived from the brain, while the anterior pituitary is not. The anterior pituitary is derived from the brain, while the posterior pituitary is not. X The pituitary is a compound endocrine gland. Each half of the pituitary secretes one important hormone. X Some pituitary hormones have sex-specific effects. If there is too little iodine in the diet, the pituitary can grow into a goiter. X The pituitary is responsible for producing at least nine major hormones. 77. ACTH stands for adrenocorticotropic hormone, but it is produced in the anterior pituitary. What is the basis of this name? A. It is expressed most highly in tropical climates. B. It stimulates the adrenal cortex. C. It signals to the kidneys to reabsorb water. D. It stimulates appetite by acting on the cerebral cortex. 78. What stimulates the production of adrenocorticotropic hormone (ACTH)? A. GHRH B. CRH C. CIH D. ADH 79. Which are important components of the communication between the hypothalamus and anterior pituitary? (Check all that apply.) ADH and oxytocin X capillaries fibrous pituitary stalk X GHIH, PIF, and MIH X hormones X hypothalamohypophyseal portal system transport of hormones down axons X TRH, CRH, GnRH, and GHRH 80. Which are important components of the communication between the hypothalamus and posterior pituitary? (Check all that apply.) X ADH and oxytocin capillaries X fibrous pituitary stalk GHIH, PIF, and MIH X hormones hypothalamohypophyseal portal system X transport of hormones down axons TRH, CRH, GnRH, and GHRH 81. Your doctor returns with your lab results and looks grave. “I’m afraid,” she says, “you seem to have a problem with your hypophysis.” What do you say? A. “Doctor, right now I’m interested in results, not testable ideas about how things work!” B. “Well, that’s only responsible for one or two hormones, so it should be easy to treat.” C. “Am I going to need insulin shots?” D. “Is it the posterior? Neurological problems run in my family.” Cushing's syndrome causes weight gain, excessive sweating, and muscle weakness. It can result from either the production of abnormally high levels of cortisol by the body, or overuse of glucocorticosteroid drugs. You are an endocrinologist working with a Cushing's syndrome patient. She is on hormonal birth control pills and takes an occasional ibuprofen. 82. What lab tests would you order to find the source of the problem? A. cortisol, LH, and FSH B. None -- the birth control pills include corticosteroid hormones. She should switch to another form of birth control. C. cortisol, ACTH, and CRH D. cortisol and ACTH 83. Results from lab tests reveal abnormally high endogenous hormone levels, and you suspect a possible tumor. What regions will you ask the MRI technician to focus on? A. hypothalamus, anterior pituitary, and adrenal cortex B. adrenal cortex only C. hypothalamus and posterior pituitary D. anterior and posterior pituitary 84. You have a pediatric patient with signs of excessive growth, or gigantism. Initial tests reveal abnormally high levels of growth hormone. What other hormone disruptions might be the primary cause of his symptoms? (Check all that apply.) high GHIH X low GHIH X high GHRH low GHRH high GnRH low GnRH 85. A middle-aged female patient complains of changes to her face. Her cheekbones and eyebrows have become more prominent, and her jaw is growing. You suspect acromegaly, a disorder caused by excessive growth hormone. She had no growth abnormalities in her youth. What do you suspect is the most likely cause of her problem? A. a tumour of the pineal gland B. congenital mutation of the GH regulatory region C. a tumour of the anterior pituitary D. a tumour of the posterior pituitary E. menopause 86. Increased use of MRI technology has resulted in a surprising medical dilemma. Scans used for early detection of tumors often reveal benign internal growths that would otherwise have gone undetected, called incidentalomas. This creates the problem of determining whether the growths are harmful without performing needless surgeries. Assume that you have found a pituitary incidentaloma in one of your patients. What blood tests could you perform to determine whether it is causing dysfunction? (Check all that apply.) X adrenococrticotropic hormone (ACTH) X antidiuretic hormone (ADH) calcitonin X follicle stimulating hormone (FSH) glucagon X growth hormone (GH) insulin X luteinizing hormone (LH) X melanocyte-stimulating hormone (MSH) melatonin X oxytocin X prolactin (PRL) X thyroid-stimulating hormone (TSH) thyroxine 87. Your friend is concerned because although she has never been pregnant, her breasts are producing milk. When she went to the doctor, he sent her to the MRI lab for a brain scan. She was too embarrassed to ask the doctor why. What do you tell her? A. "They were actually scanning your thyroid gland, not your brain. Are you getting enough iodine in your diet?" B. "The MRI technician must have mixed up your patient records. He should have done a thoracic scan instead." C. "LH and FSH from the posterior pituitary stimulate milk production. The doctor wants to rule out excess levels of estrogen in the pituitary stalk." D. "Prolactin from the anterior pituitary stimulates milk production. The doctor probably wants to rule out a pituitary tumor." 88. True or False: The thyroid requires negative feedback to prevent overgrowth. TRUE The resulting overgrowth is called a goiter. 89. True or False: If Ca+2 levels in the blood are too low, hormones from the parathyroid gland can induce bones to dissolve and release calcium. TRUE 90. Blood glucose levels must be carefully maintained within a narrow window. What endocrine organ is responsible for this task? A. pancreas B. pineal gland C. pituitary gland D. parathyroid gland 91. What strategy does the body use for blood glucose homeostasis? A. release of hormones by the brain B. energy stores in bone allow for quick glucose release C. glucose usage by the brain is shut down when necessary D. the action of two antagonistic effectors 92. When a diabetic injects insulin, what does this accomplish? A. promotes glucose release from cells, raising blood glucose B. promotes uptake of glucose into cells, lowering blood glucose C. promotes hydrolysis of glycogen, raising blood glucose D. improves the ability of the body to sense blood glucose levels 93. Estrogen and testosterone are steroid hormones that differ by only a few atoms. The different receptor binding properties those differences confer allow the hormones to do what during development? A. Cause differential maturation of the prefrontal cortex during late adolescence. B. Cause a slight increase in muscle mass in females versus males. C. Make females naturally attracted to pink and males to blue. D. Control development of all sex-specific physical traits including sex organs, body hair, and breasts. 94. Atrial natriuretic hormone (ANH) stimulates the kidneys to excrete salt and water in the urine. Why does it make sense that the heart is the source of ANH? A. The heart is a major endocrine organ that releases several hormones. B. ANH helps regulate blood pressure, which is sensed by distension of the heart atria. C. The heart is conveniently close to the hypothalamus. D. Secretion of hormones by the heart makes it easy for them to enter the bloodstream. 95. What nonendocrine organ produces erythropoietin, which stimulates red blood cell production in the bone marrow? A. kidneys B. skin C. heart D. pituitary E. liver Many insecticides target insect hormones such as juvenile hormone or ecdysone to disrupt development. This is preferable to simply killing the insect with more broadly-acting toxins. 96. Your cat has fleas! You look at the ingredients in the flea control ointment, and see that it contains methoprene, a juvenile hormone analog. What does this accomplish? A. prevents flea larvae from maturing B. kills juvenile fleas C. reinvigorates the cat's immune system, to repel the fleas D. forces juvenile fleas to mature early, disrupting development of reproductive organs 97. What is a benefit of targeting hormone signaling rather than a generic cellular housekeeping process in insects? A. It is water soluble. B. It works very quickly. C. It disrupts the entire population in the area. D. It is specific to insects and less toxic to mammals. 98. G protein coupled receptors are a large class of receptors that bind hormones as well as other signaling molecules, neurotransmitters, and even odorants. They are a major target of pharmaceutical drugs for various disorders. Typically, the drug acts as an antagonist, blocking the binding site on the GPCR. Why is the receptor a more attractive drug target than the G protein activated downstream? (Check all that apply.) Interfering with the G protein would require getting a drug across several membranes into the cell's nucleus. X Since GPCRs are so widespread and perform many roles in the body, targeting G proteins used by multiple GPCRs would create side-effects beyond the targeted physiological function. X Since the GPCRs bind a variety of unique ligands, their structures are varied enough to be able to create specific targeted drugs for certain pathways. X It is easier to target molecules from the outside of the cell membrane, because getting drugs into the cell can be difficult. The "G-protein" is not really a protein but just a GTP molecules, which is too small to be a possible drug target. 99. Beta-blockers are a class of drugs that target beta-adrenergic receptors, blocking the action of epinephrine and norepinephrine. These drugs are used to treat heart conditions, hypertension, glaucoma, and migraine headaches and are also used off-label to alleviate stage fright in performers. Since the beta-adrenergic receptor is a GPCR, what intracellular event is prevented by the use of a beta-blocker? A. a protein kinase cascade, initiated by the receptor's intracellular kinase domain B. release of an activated G protein subunit bound to GTP C. transport of the receptor-hormone complex into the cell and the nucleus, where it would regulate transcription D. a protein phosphatase cascade, initiated by the receptor's intracellular phosphatase domain 100. Ultimately, the signal from a peptide hormone is typically conveyed to the effectors in the cell via the transfer of what molecular moiety? A. nucleotide B. NO (nitric oxide) C. H2O D. phosphate group E. amino acid Whether the peptide hormone receptor is a receptor kinase or a GPCR, the intracellular cascade often involves one or a series of kinases each transferring a phosphate group onto their targets. 101. To exert its effects on gene regulation in a cell, a steroid hormone must interact with a variety of proteins and cell structures along the way. Check the box for each interaction partner. X carrier protein microtubules X plasma membrane X hormone receptor X nuclear membrane GPCR Chapter 55: Ecology of Individuals and Populations 1. Populations of endotherms that live in colder climates tend to have shorter ears and limbs than populations of the same species in warm climates. This is called Rule. A. Allen's B. the K-Selected C. the r-Selected D. Edward's 2. The rate at which a population of a given species will increase when no limits are placed on its rate of growth is called its A. maximum growth. B. carrying capacity. C. biotic potential. D. optimal growth. 3. The size at which a population stabilizes in a particular place is defined as the for that species. A. growth potential B. optimum C. range D. carrying capacity 4. Life history adaptations of populations are characterized by an early age of first reproduction and short maturation time and life span. A. r-selected B. K-selected 5. Species that have a delayed reproductive stage, are competing for limited resources, and have smaller numbers of slowly maturing large offspring show adaptations. A. K-selected B. r-selected 6. The statistical study of populations including sex ratio, age structure, and predicting growth rates is called . A. ethology B. demography C. population genetics D. biometrics 7. A group of population members all of the same age is called a . A. deme B. species C. cohort D. tribe 8. Organisms such as lizards that need to maintain body temperature through external means do so by A. shivering to produce heat. B. sunbathing. C. swimming in geothermal pools. D. running frequently to warm up. E. eating large meals to provide calories for heat production. 9. What is the life history adaptation called where organisms produce offspring several times over many seasons? A. semelparity B. iteroparity C. biparity D. polyparity E. alloparity 10. Effects that are dependent on the size of the population and regulate the growth of populations are called effects. A. K-related B. density-independent C. environmental resistance D. density-dependent E. demographic 11. The trade-off between investments in current reproduction and in growth that promotes future reproduction is referred to as the total cost of what? A. adaptation B. selection C. reproduction D. genetic change E. fitness 12. A small group of mice are released on an island without mice but with abundant food for mice and no predators. Initially, the growth of the mouse population will be limited mainly by what? A. the carrying capacity B. the biotic potential C. only density-dependent factors D. only density-independent factors 13. A small group of mice were released on an island. The island previously had no mice on it, but had abundant food and no predators. After several years of growth, the size of the new island population stabilizes. However, at this point a hurricane drastically reduces the population. How would you describe the situation? A. The biotic potential of the population has been reduced. B. The new population size is a result of density-dependent regulation. C. The new population size is a result of density-independent regulation. D. The island mouse population can now act as a sink metapopulation. E. The island mouse population can now act as a source metapopulation. 14. Demographic studies include all of the following except what? A. age structure B. growth rates C. mortality and survivorship curves D. sex ratio E. measurements of interspecific competition 15. Clumped or patched populations that undergo local periodic extinction and recolonization are called what? A. randomly spaced populations B. uniformly spaced populations C. metapopulations D. over-sized populations E. endangered populations 16. The biotic potential representing growth without limits at its maximal rate is indicated by what symbol? A. ri B. N C. K D. dN/dt E. N/K 17. In the sigmoid growth curve, the number of individuals at any one time is indicated by what symbol? A. ri B. N C. K D. dN/dt E. N/K 18. In the sigmoid growth curve, the carrying capacity of the environment is indicated by what symbol? A. ri B. N C. K D. dN/dt E. N/K 19. The range of a population A. is very fluid and changes frequently in a random fashion. B. is stable and almost never changes. C. only changes after a disaster has wiped out a former range. D. changes over time due to external events. E. only changes due to iteroparity. 20. Which of the following describe density-dependent factors? X They act to regulate population growth. X They are especially important in K-selected populations. They can affect growth rates but not population size. X They can affect birth rates or death rates. They do not involve biological interactions. X One example would be intraspecific competition for resources. 21. Which is an example of a density-independent factor on a population? A. weather B. nesting sites C. intraspecific competition D. amount of prey E. predation 22. What do we call populations of organisms that are usually near the carrying capacity? A. K-selected B. r-selected C. predators D. parasites E. perennials When resources are limited, the cost of reproduction often will be very high. Consequently, selection will favor individuals that can compete effectively and utilize resources efficiently. Such adaptations often come at the cost of lowered reproductive rates. Such populations are termed K-selected because they are adapted to thrive when the population is near its carrying capacity (K). Examples of K-selected species include coconut palms, whooping cranes, whales, and humans. By contrast, in populations far below the carrying capacity, resources may be abundant. Costs of reproduction are low, and selection favors those individuals that can produce the maximum number of offspring. Selection here favors individuals with the highest reproductive rates; such populations are termed r-selected. Examples of organisms displaying r-selected life history adaptations include dandelions, aphids, mice, and cockroaches. Most natural populations show life history adaptations that exist along a continuum ranging from completely r-selected traits to completely K-selected traits. 23. What are characteristics of r-selected populations? (Check all that apply.) X early age of first reproduction late age of first reproduction small brood size X large brood size X little or no parental care extensive parental care X short generation time long generation time type I survivorship curves X type III survivorship curves 24. Which of the following is an example of a species with r-selected adaptations? A. house fly B. blue whale C. gorilla D. horse E. cockroaches 25. Which is an example of a species with K-selected adaptations? A. dandelion B. fruit fly C. herring D. bobcat 26. A species colonizes an island, goes through a phase of exponential growth, and reaches the carrying capacity of the island. At what stage is the growth rate slowest? A. initial colonization B. exponential phase C. approaching K D. after reaching K 27. In the logistic growth model, as the number of individuals in the population (N) approaches the carrying capacity (K), what happens to the intrinsic rate of growth (ri)? A. It will be affected by an increased birth rate. B. It will be affected by a decreased death rate from predation. C. It will be affected by increased competition with other species. D. It will be affected by increased competition within the species. E. It will not change. 28. There are three aspects of entire populations that are important and often studied. Select the best choice from the ones listed. A. a population's range, the dispersal of individuals within the range, and the size that the population attains B. a population's range, the amount of food available within the range, and the size that the population attains C. a population's range, the parental care received by each offspring within the population, and the size that the population attains D. a population's range, the size home range of an individual in the population, and the parental care expended for each offspring 29. During the late 1800s, cattle egrets arrived in South America from Africa and began to colonize. Their range has expanded dramatically over the years. Why were they able to do this? A. The habitats that they left in Africa were not suitable for any further colonization; thus, they were forced to emigrate. B. The habitats that they encountered in South America were suitable to them and unoccupied. C. There were abundant cattle for the birds to gather around in South America; furthermore, various animals that the egrets had lived around in Africa had become quite scarce because of over hunting and poaching, causing the birds to extend their range. D. The food resources in South America were far superior to those in Africa, allowing the egrets more opportunity to grow and reproduce and ultimately expand their range. The range of a population may expand either because environmental changes improve the suitability of a new region, or because a chance event allows a species to cross a barrier into a new suitable region. The relative quality of the old range has no bearing on whether the population will expand into the new region. In the example of the cattle egret, some change crossing of the Atlantic allowed the species to arrive in South America in the 1880s, and the population has gradually expanded both northward and southward into all suitable regions. 30. Populations have three basic types of dispersal patterns—clumped, random, and uniform. What situation supports random distributions? A. When individuals of the populations do not interact strongly with one another. B. When individuals of the population are in competition for resources. C. When there is an even distribution of resources. D. When individuals of the population display social interactions. 31. Which is an example of a clumped distribution of a population? A. A tree species releases a poison to defend the soil around its roots, resulting in trees spaced evenly like a grid. B. Antelope travel in herds. C. Two male hummingbirds have a confrontation on the border between their territories. D. Multiple species join each other at the waterhole to drink. Social interactions between members of a species can lead to clumped distributions. Many species live and move around in large groups (such as flocks, herds, or prides). These groupings can provide many advantages, including defense against predators, decreased energy cost of moving through air and water, and access to the knowledge of all group members. 32. Many times species are composed of networks of distinct populations called metapopulations. When do metapopulations occur? A. When a population is large and uniformly distributed. B. When a population in poor habitat continually sends out dispersers to bolster populations in better habitats. C. When a population in a better habitat does not send out colonizers into less suitable habitats. D. When suitable habitat is patchily distributed and separated by areas of unsuitable habitat. 33. Which factor increases the likelihood of population extinction? A. large population size B. isolation of a population from density-independent effects C. isolation of a population from sources of immigrants D. high resource availability E. high genetic diversity 34. Which of the following statements is true about the graph? A. Oysters live longer than Hydra. B. Hydra and humans have parallel life spans. C. Humans and oysters have similar life spans. D. Humans have low mortality rates early in life. E. Oysters have high mortality rates late in life. 35. What often describes organisms with a Type III life history? A. K-selected B. r-selected C. idiopathic D. at their carrying capacity E. subject to low predation rates 36. What species would probably have a Type III life history? A. whale B. coconut palm C. lion D. gorilla E. pine tree 37. What is the Allee effect? A. when fitness increases independently of population size B. when fitness and growth rates increase with decreasing population size C. when fitness and growth rates increase with increasing population size D. when growth rates increase independently of population size 38. What are population pyramids used to show? A. death rates B. birth rates C. competition D. age composition of a population E. the carrying capacity 39. Which of the following apply to the demography of human populations today? (Check all that apply.) Our ecological footprint is decreasing. X We are using distributed resources disproportionately—less than 20% of us use over 80% of the energy. We have plenty of resources still untouched. X We use almost half of the Earth's land. X We use over half of all renewable fresh water sources. 40. What kind of population spacing would you expect for a species that is strongly territorial? A. uniform B. clumped C. random D. uniform or clumped depending on whether it is a source or sink metapopulation E. uniform or clumped depending on the life history strategy of the species 41. Which of the following are environmental factors that determine where an organism can live? (Check all that apply.) intraspecific competition predation prey availability X soil type X sunlight X temperature X water 42. You construct a life table for a plant species and find that in all cases about the same proportion of the cohort survive to the beginning of the next time interval. What would you call this? A. a Type I survivorship curve B. a Type II survivorship curve C. a Type III survivorship curve D. a semelparous life history adaptation E. populations regulated by density-independent events 43. Allen's rule states that mammals from colder climates have shorter ears and appendages than individuals of the same species from warmer areas. The related Bergman's rule states that mammal body size varies with latitude, with larger body size in populations located closer to the poles and smaller body size in populations located closer to the equator. What can explain both of these rules? A. Larger bodies can store proportionally more fat as insulation than smaller bodies. B. Smaller bodies can store proportionally more fat as insulation than larger bodies. C. Heat loss increases as the surface area to volume ratio of the animal increases. D. Larger bodies have a proportionally larger surface area, and heat radiation varies with volume. E. Smaller bodies have a proportionally smaller surface area, and heat radiation increases with surface area. 44. Which of the following describe source-sink metapopulations? X Sink populations near source populations are less likely to go extinct. Sink populations without access to immigrants from source populations are less likely to go extinct. X Source populations usually occupy better habitats. X Source populations are less likely to go extinct. X The emigration rate from source populations exceeds that from sink populations. 45. Parental care of the young is usually associated with species with a Type survivorship curve. A. I B. II C. III D. I or II (depending on the sex ratio) E. I or III (depending on whether the reproductive strategy is iteroparous or semelparous) 46. Which one of the following expressions from the logistic equation (dN/dt = rN ((K - N)/K) represents the proportion of unused resources remaining for use by the population? A. the carrying capacity (K) B. the population size (N) C. the biotic potential (rN) D. (K - N)/K E. the growth rate (dN/dt) 47. Under which condition will a population experience growth? A. when N equals K B. when N is less than K C. when dN/dt equals zero D. when r equals zero E. when N equals zero 48. What is the most reasonable conclusion that can be made from data graphed in the figure? A. After high mortality early in life, survivorship becomes constant for the rest of life. B. Survivorship is constant throughout life. C. After low mortality early in life, mortality increases to a constant rate for the rest of life. D. After following a constant rate early in life, mortality becomes low late in life. 49. Most density-dependent factors show negative feedback relative to population size. Which one of the following is more likely to show positive feedback relative to population size? A. competition in small populations B. competition in large populations C. fecundity in small populations D. fecundity in large populations E. mortality in large populations 50. The figure included shows the body temperature of lizards versus air temperature in two different habitats—open and shaded forest. Which one of the following conclusions is best supported by these data. A. Lizards are more active in open habitats. B. Lizards sunbathe more in open habitats. C. Lizards in shaded forest habitats eat more to maintain their temperature. D. The body temperature of lizards is more constant in open than in shaded forest habitats. E. The negative effect of air temperature on body temperature is less in shaded forest habitats because the temperature varies less there. 51. Select the description of a population pyramid for a population that has the largest potential for exponential growth. A. broad at the base, narrow at the top, with more females than males B. broad at the base, narrow at the top, with more males than females C. uniform at all age classes, with more females than males D. broad at the top, narrow at the base, with more males than females E. broad at the top, narrow at the base, with more females than males 52. Many organisms have adaptations to avoid freezing temperatures, including behaviors like migration, features like thick fur, and molecular features like antifreeze molecules. However, some organisms have a adaptation called ice nucleation, in which special proteins help ice crystals to form at warmer temperatures. What organism would be most likely to benefit from such an adaptation? A. a bacterial pathogen that feeds off damaged plants B. a saltwater fish that lives in coral reefs C. a bird species that migrates long distances D. a species of yeast used in brewing beer In fact, the bacterium Pseudomonas syringae is thought to be responsible for much frost damage in plants, due to its special ice nucleation-active (INA) proteins that promote formation of ice crystals at higher than normal temperatures. The resulting frost damage is thought to benefit the pathogen by releasing nutrients from the broken tissues of the plant. In 1987, an "ice-minus" genetically-engineered strain was created to attempt to improve crop yields, but was too controversial to develop commercially. The ice-plus strain is currently used to improve snow production at ski resorts, and some have suggested that the bacterium may play an important role in our normal weather patterns as well! 53. What environmental challenge is posed to young plants by an old growth forest? A. low humidity B. high temperatures C. low light D. thick leaf litter 54. You and your friends have just stepped off the plane in Cusco, Peru (11,200 ft). Your friends are so eager to see Machu Picchu they want to start off hiking the Inca Trail right away and not waste any time. How do you respond? A. "Sounds good, but we had better pack extra water." B. "Sounds good, but we had better bring our warmest clothes." C. "No problem -- we had the whole plane ride to acclimatize to the high altitude." D. “Waiting a few days might be smart, since it takes time to acclimate to high altitude.” 55. A zoo in Southern California has been successfully breeding arctic wolves, and they want to exchange some of their extra wolves for kangaroos from a zoo in southwestern Australia. Since it is July, it is quite cold in the other zoo compared to Southern California. What discussion might the zookeepers have about transferring the wolves? A. Since Arctic wolves are well-adapted to cold, this should pose no problems. B. After several generations in Southern California, the wolves have lost their adaptive genes for cold temperatures. C. After growing up in Southern California, the individual wolves have lost their ability to adapt to cold temperatures, but their progeny will still get the cold-adapted genes. D. The wolves have not grown their cold weather coat. 56. There are not many species of fish living deep in the oceans, but the deep-sea fish occasionally pulled up by fisherman have been bizarre creatures, such as the deep-sea anglerfish with its giant fangs and bioluminescent lure. Why have these fascinating deep-sea fish never been shown live on exhibit in an aquarium? A. Their diet is too specialized. B. They are adapted to cold temperatures and total darkness. C. They might frighten visitors. D. They are adapted for high pressure, and will die at low pressure. The most difficult requirement of deep-sea fish is high pressure. They are adapted to the extreme pressure of the deep sea, and without that pressure their cells and tissues will not function properly and they will die. Bringing a deep-sea fish up to the surface and constantly maintaining the appropriate pressure is a major challenge to ever displaying these animals at the surface. 57. You and your friend are on vacation at Yellowstone Park. The plaque in front of a sulfur hot spring reads: “This sulfur spring is 80C, has a pH of 2, and contains large amounts of sulfur which are oxidized by Sulfolobus species of archaea.” Your friend is excited. “Boy, if those microbes are that tough they should be easy to grow in the lab. I think I’ll work with those for my summer project.” What do you think? A. You agree, and decide to partner with your friend to work on this organism as well. B. A species adapted to such extreme conditions would probably not be easy to culture in the very different conditions of a lab. C. The sulfur spring is pretty extreme, but after a few days the individual archaea should have adapted to lab culture conditions. D. The sulfur spring is pretty extreme, but after a few generations the species should have adapted to the lab culture conditions. 58. You are earning your Ph.D. in marine biology, studying the microbial ocean community. This year, you are shocked to discover that one of your study sites has become contaminated due to an industrial waste water pipe. This dumping is drastically raising the temperature and lowering the salinity of the water. What do you expect to find in your survey of the population, and what might you find in a survey years from now? A. Most species will be wiped out this year, and in subsequent years even fewer will remain. B. The individuals will turn on genes to adapt to these changes, and the populations will end up largely the same this year and following years. C. Many species will have decreased or disappeared from this site, but later some species may increase in population if they acquire adaptive mutations. Also you may see some new species expand into the site. D. Most species will be wiped out this year, and it will take millions of years for new species to evolve -- until then the site will be devoid of life. 59. What factor allows bacteria species to adapt to environmental changes faster than fish? A. shorter generation time B. smaller size C. smaller nutrient needs D. lack of a nuclear membrane 60. Kettle ponds derive from a hole left by a retreating glacier that is subsequently filled by rainwater. A certain region has a series of disconnected kettle ponds, but occasionally heavy rains flood them and create streams. Which type of animal is most likely to be present in this area as a metapopulation? A. a fish B. an aquatic bird C. a mosquito D. a dragonfly 61. A farmer’s land includes a wilderness area that is home to an endangered species of bird. He wants to sell the land to a developer, and you have been hired to evaluate the environmental concerns. “This little patch of land is not crucial for the bird species!” the farmer says, “In fact, my land is surrounded by several other wilderness patches that also have that bird.” You reply: “It is possible that your land acts as the and the other areas are . If that's the case, development would be devastating to the population." A. sink; sources B. source; sinks C. population; metapopulation D. metapopulation; population 62. An underwater volcano has erupted, and as the lava cools a new island has formed. Which types of animal species are likely to be among the first to colonize the island? annelids X bats X birds X insects snails snakes tapirs tortoises 63. You work for a successful company that sells maternity clothes. They have asked you to evaluate whether Australia or Singapore would be a better country to expand into with new stores. What keyword will help you find the data you need to make this evaluation? A. range expansion B. dispersal C. metapopulation D. demographics 64. Two populations have very similar birth rates and death rates, but the change in the growth of the populations is quite different. What other factors might account for the difference? (Check all that apply.) X age structure primary food source X generation time X immigration X sex ratio 65. Within a genus that normally produces large numbers of offspring, one particular species evolves maternal care behavior. What do you predict will happen to the survivorship curve? A. No change is likely. B. It may change from type I to type III. C. It may change from type III to type I. D. It may change from type I to type II. 66. Which organism is most likely to have a type III survivorship curve? A. an oak tree B. a domestic dog C. corn in a farmer's field D. Paramecium 67. An unscrupulous dog breeder has been selecting for dogs that have very large litters, with the goal of maximizing profits. Why might this strategy be counterproductive? A. It will cost too much money to feed all those puppies. B. Litter size is not influenced by genetics. C. The puppies from large litters are likely to be malnourished, and the mother will have few resources to devote to future litters. D. Intrauterine cannibalism will reduce the number of puppies. 68. A graduate student is studying the feeding behavior of a small octopus that must be laboriously collected from the shores of a remote island. Her intern wishes to do experiments to look at the octopus’s mating behavior. Why might the graduate student veto this idea? A. The octopus has semelparous reproduction, and will eat its mate. B. The octopus has iteroparous reproduction, and will eat its mate. C. The octopus has semelparous reproduction, and will die after laying one batch of eggs. D. The octopus has iteroparous reproduction, and will die after laying one batch of eggs. 69. A rancher is suspected of shooting wolves near Yellowstone National Park, and you have been enlisted to go have a chat with him. The rancher doesn’t see any benefit to having the wolves there. What do you say? A. "If there are no predators like wolves, just one pair of jackrabbits can quickly create a population with explosive, logistic growth. Wolves help reduce the fecundity rate, keeping the population at a reasonable level." B. "If there are no predators like wolves, just one pair of jackrabbits can quickly create a population with explosive, exponential growth. Wolves help reduce the carrying capacity, keeping the population at a reasonable level." C. "If there are no predators like wolves, just one pair of jackrabbits can quickly create a metapopulation. Wolves help reduce the biotic potential, keeping the population at a reasonable level." D. "If there are no predators like wolves, just one pair of jackrabbits can quickly create a population with explosive, density- dependent growth. Wolves help reduce the age structure, keeping the population at a reasonable level." 70. A population of squirrels is in a phase of exponential growth. What events would act to slow this growth? (Check all that apply.) X a forest fire that destroys habitat X logging of nut trees X growth of the hawk population heavy rainfall, resulting in larger seed and nut harvest X increased population that is outstripping food supply less competition from a diminished population of ground squirrels 71. A developing nation with a rapidly increasing population is reaching its carrying capacity. What change(s) could allow it to increase its carrying capacity? X a cultural shift to eating less meat education for women X greater yield from agricultural crops increased availability of birth control X water treatment plants The carrying capacity is the maximum number of individuals that a given environment can support. Anything that increases the food or water resources available to support people would increase carrying capacity, such as improved crop yields, a shift to vegetarian diets (which require far less land per calorie of food produced), and water treatment plants to produce clean water. 72. You are working in an aquarium that has been breeding an endangered species of fish for return into the wild. You have a batch of progeny from the first generation of breeding. The staff are discussing whether to return those fish to an area that has been completely depleted, or add them to a partially depleted population. “Consider the Allee effect,” you say A. “They may survive better in a large school, and will have an easier time finding mates.” B. "They may do better in the empty area without any competition for food." C. "They may do better in the empty area without any competition for territory." D. "The area with the other fish may reduce their exposure to disease." 73. This summer, you are returning to the research station in Costa Rica to follow up on the population of butterflies you have been studying. You are disappointed to find that there are fewer this year than last. In fact, at dinner your friends studying frogs and birds are complaining about the same thing. What type of influence do you suspect? A. a density-independent effect like an introduced predator B. a density-dependent effect like environmental disruption C. a density-independent effect like environmental disruption D. a density-dependent effect like an introduced predator 74. True or False: An r-selected species is more likely to become an invasive pest than a K-selected species. TRUE 75. What would be the best approach to curbing excessive human population growth? A. lowering birth rates through education and family planning B. a return to historical health care practices C. encouraging women to postpone children till after age 35 D. embracing traditional agricultural practices 76. What age group on a population pyramid would you examine to predict the population in 25 years? A. 0-5 B. 10-15 C. 20-25 D. 25-30 77. A group of international students is having a heated discussion in the dining hall. “Your country needs to get its population under control!” asserts a student from the United States. “Perhaps,” replies the Indian student, “but your country .” A. has the highest per capita resource consumption, ten times greater than mine B. also has a big population overgrowth problem C. has a much higher birth rate D. has the lowest infant mortality rate [Show More]

Last updated: 1 year ago

Preview 1 out of 54 pages

Add to cart

Instant download

document-preview

Buy this document to get the full access instantly

Instant Download Access after purchase

Add to cart

Instant download

Reviews( 0 )

$14.00

Add to cart

Instant download

Can't find what you want? Try our AI powered Search

OR

REQUEST DOCUMENT
38
0

Document information


Connected school, study & course


About the document


Uploaded On

May 09, 2021

Number of pages

54

Written in

Seller


seller-icon
Goodluck Academia

Member since 3 years

164 Documents Sold


Additional information

This document has been written for:

Uploaded

May 09, 2021

Downloads

 0

Views

 38

Document Keyword Tags

More From Goodluck Academia

View all Goodluck Academia's documents »

$14.00
What is Browsegrades

In Browsegrades, a student can earn by offering help to other student. Students can help other students with materials by upploading their notes and earn money.

We are here to help

We're available through e-mail, Twitter, Facebook, and live chat.
 FAQ
 Questions? Leave a message!

Follow us on
 Twitter

Copyright © Browsegrades · High quality services·